for display only
Big Blue Interactive The Corner Forum  
Back to the Corner

Archived Thread

NFT: Dumb Economics Question

Reb8thVA : 5/26/2015 12:45 pm
So the Dow was down triple digits this morning if I understand correctly because of improving economic news fueling fears of an interest rate hike. In emphasizing 401Ks, thrift savings plans and the like we probably have the largest class of people in the US that are stock holders than ever before. So have we gotten to the point where you root for bad economic news so that your stock holdings go up at the expense of the overall health of the economy?

Does this system we seem to have created make sense to anyone any more? It just seems like a house of cards.
If there is bad economic news  
Wellington : 5/26/2015 12:51 pm : link
Your stocks will also go down.

All that this proves is that you need to consistently and continuously contribute to your 401k, IRA, 529, or whatever stock based investments you are holding. By doing so, Dollar Cost Averaging, you will invest when stocks are cheap and even out when they are expensive.

Anecdotally, one of the smartest moves I made was in the heart of the recent recession I increased my contribution amount and let it ride out until late last year at the same number. I was buying more stocks at a cheaper price and when they eventually came back up my retirement portfolio came up with it.

I wish I had been smart enough to buy the corresponding funds in taxable accounts. Would have made a pretty penny.
Well, it's pretty much true for any tradeable commodity, with  
kicker : 5/26/2015 12:53 pm : link
the stock market being another iteration of a wealth accumulation mechanism.

It's expectations and animal spirits. A lot of people are close to being "market makers", where their words are given undue influence.

If we didn't have a stock market, it would be likely we would see a return to earlier methods of preserving and keeping wealth; land speculation, finery, gold, etc. Those were subject to similar boom-bust-boom cycles.
And the "good" economic news leading to downward biased  
kicker : 5/26/2015 12:54 pm : link
stocks is a rare phenomenon. It happens when there has been artificial pressures in the market (such as Fed intervention) that have not created interest rates that are close to market clearing.

An interest rate hike isn't inherently "good", as it can slow down growth, increase borrowing costs, and lead to problems in the labor market.
RE: Well, it's pretty much true for any tradeable commodity, with  
River Mike : 5/26/2015 1:02 pm : link
In comment 12300689 kicker said:
Quote:
the stock market being another iteration of a wealth accumulation mechanism.

It's expectations and animal spirits. A lot of people are close to being "market makers", where their words are given undue influence.

If we didn't have a stock market, it would be likely we would see a return to earlier methods of preserving and keeping wealth; land speculation, finery, gold, etc. Those were subject to similar boom-bust-boom cycles.


Not to mention, tulips
The causal links between the economy and the market  
SwirlingEddie : 5/26/2015 1:03 pm : link
seem more tenuous than ever.
Central bank intervention  
Rob in CT/NYC : 5/26/2015 1:09 pm : link
over the past six years has had a distortive effect on asset prices - equities, commodities, bonds. The recent perversion - good news for the economy is bad for the market - is the simply the reverse of what we have seen in the recent past.

Asset prices have becoming detached from underlying drivers of value.
It's the bizarre logic of the markets these days  
RB^2 : 5/26/2015 1:11 pm : link
You can thank the central banks for this. On the other hand, without them the Dow is nowhere near 18K. I won't even get into the credit "markets".
Rob...  
Reb8thVA : 5/26/2015 1:12 pm : link
your point is what I was trying to say, I just couldn't articulate it correctly.
If anything, Volcker may have been the worst thing to  
kicker : 5/26/2015 1:17 pm : link
happen to the Fed, ever.

He showcased that a strong will can bend the economy in the short-run. The Fed has taken this further and uses it as prima facie evidence that their actions are justified.
I just dont understand the OP  
Deej : 5/26/2015 1:17 pm : link
Generally speaking the broad stock market goes UP on good economic news. So why would you need to root against it?
2016. the stock market will take a big hit on the downside.  
charlito : 5/26/2015 1:20 pm : link
Make sure you increase your contribution in your 401k even more. We're overdue for a correction.Max out Roth ira (the best of the ira's) traditional ira if you want a couple extra g ' s extra on your tax return. Your Roth ira won't be taxed when you're of age to take out.
interest rates  
giantfan2000 : 5/26/2015 1:41 pm : link
Quote:
Asset prices have becoming detached from underlying drivers of value.


umm wrong

historic PE is 20.55
a bit higher than historic average (15.54) but no way "Detached from underlying drivers of value"

that said
if Interest rates do get raised in summer as anticipated
then Stock market will be considered very overvalued and expect a sharp pull back.


When the Fed purchases securities, and distorts the market,  
kicker : 5/26/2015 1:49 pm : link
the returns and risk calculations are inherently different. The stock prices no longer correspond to long held fundamentald, even if there is a faint correlation.

The correct answer is not what they are now, but what they would be in the absence of extreme intervention by central banks.
Good News is Bad News Bad News is Good News....  
Dinger : 5/26/2015 1:54 pm : link
Its been going on for as long as I've been around the market and thats 20+ years. There are stretches when good news is actually good news too.

But you have to consider the source. Media has to be able to explain EVERYTHING so today its consumer confidence. Tomorrow it'll be up and its Apple buying General Motors. For any other traders on the forum they know its simple....today there are more sellers than buyers in the market as a whole (or is that hole). As we get further into the summer, days like this happen more frequently on Fridays and Mondays(today being a quasi monday).

I do agree with your premise that EVERYONE is in a 401k and thus more money than ever is in the Stock market. Part of me has this doomsday mentality that says take half of it out and put it under my mattress!
explore this issue here  
idiotsavant : 5/26/2015 1:57 pm : link
https://www.youtube.com/watch?v=d0nERTFo-Sk

of course, I don't know your answer, however, these two gents are highly regarded and engaging in a very enlightening debate.

It's about the correlation between policy, inflation, actual growth.  
manh george : 5/26/2015 2:07 pm : link
Optimal for the stock market is an environment where growth can continue without inflation, so that the Fed doesn't have to take away the cookie jar.

On the other extreme is stagflation, whereby the Fed has to tighten even though the economy isn't all that impressive.

As a continuing believer in the benefits of accelerating technological change for owners, I think that we can see very modest inflation and decent growth continue a very long time without the Fed having to do much. I get nervous about the "out years" when technology supersedes demand for labor. I also worry if central bankers do more than they really need to, to protect against inflation.
Manh  
Bill2 : 5/26/2015 2:17 pm : link
as we have discussed, any kind of investment is greatly accelerated when artificially low interest rates, set to clear the downside of past banking bubbles and bad risk management without much consequence (with the upside taken by the banking system and players)now continue to induce all kinds of mal-investment and no value to enduring differentiation and barriers to entry beyond spending on "advances"

The big risk is the future erosion of the consent of the governed. As the sparrow said: There are few and there are many. When the many get that...all kinds of trouble might brew.
the market  
giantfan2000 : 5/26/2015 2:30 pm : link
there is an old saying
the market rises on a wall of worry

this is what has happen in the past 6 years

as long as people talk about market being in a bubble we will continue to rise.
RE: the market  
Rob in CT/NYC : 5/26/2015 2:41 pm : link
In comment 12300918 giantfan2000 said:
Quote:
there is an old saying
the market rises on a wall of worry

this is what has happen in the past 6 years

as long as people talk about market being in a bubble we will continue to rise.


It's abundantly clear you have no idea what the impact of central bank intervention in the markets has been. Let me guess - you think the collapse in crude oil prices was because of excess supply?
It's not good news or bad news per se  
NNJ Tom : 5/26/2015 2:59 pm : link
Its how the news compares to the consensus of trader's thoughts on the subject.

IE Apple makes a trillion dollars, stock goes down, because the traders were looking for 2 trillion.



ha  
giantfan2000 : 5/26/2015 3:06 pm : link
Quote:
Asset prices have becoming detached from underlying drivers of value.


Let me guess you have all money in gold hidden under your bed

RE: And the  
Dunedin81 : 5/26/2015 3:13 pm : link
In comment 12300692 kicker said:
Quote:
stocks is a rare phenomenon. It happens when there has been artificial pressures in the market (such as Fed intervention) that have not created interest rates that are close to market clearing.

An interest rate hike isn't inherently "good", as it can slow down growth, increase borrowing costs, and lead to problems in the labor market.


This is the equivalent of taking the punch bowl away (should it happen, and I'll believe it when I see it). The Keynesian corollary to the stimulative approach in lean times that no one is ever comfortable with. Now the issue is whether we're in fact in anything approaching a boom, but the idea of continuing with near-zero (effectively negative) interest rates in perpetuity presents a host of problems too.
Let's disregard that raising rates right now will have some  
kicker : 5/26/2015 3:24 pm : link
less than satisfactory outcomes, especially on the local and state level, that may significantly affect short term debt. There's a number of ways that could go.

I'm of the belief we are playing a competing waltz, where central bank intervention is our way to fight to keep currency on our terms. The Russians and Chinese can utilize a media, rather than their central banks, and have been clamoring for their ticket into the currency party.

We've hedged ourself into a position that raising rates could squeeze us out of, with disastrous results.

I don't see this anymore as stimulative. I see it as combat. Gone are the days you flood the market with Confederacy currency. We have a much sharper scalpel.
Marx  
RasputinPrime : 5/26/2015 3:26 pm : link
wasn't just preaching good comedy.
Rasputin  
manh george : 5/26/2015 3:38 pm : link
Did he include a sanity clause?
RE: ha  
Rob in CT/NYC : 5/26/2015 3:49 pm : link
In comment 12300963 giantfan2000 said:
Quote:


Quote:


Asset prices have becoming detached from underlying drivers of value.



Let me guess you have all money in gold hidden under your bed


Of course not, but I am neither so deluded or indoctrinated as to be believe that the Fed's decision to inflate yet another asset bubble is investable in the long-term.
oh, you canna fool me, I know there's no Sanity Claus  
Greg from LI : 5/26/2015 3:53 pm : link
..
kicler  
Bill2 : 5/26/2015 4:19 pm : link
Check.

In the end the US will be worth .30 cents but the rest of the world will be worth .06 cents.

- Oliver, 2008
Bill2  
manh george : 5/26/2015 4:28 pm : link
So how does one tell when rates are "artificially low?" I used to think that artificially low rates would lead to excess inflation. That was the Arthur Burns story, which gave us the Volcker tightening.

I guess the new definition of too low simply means "low enough to generate asset bubbles, even if not accompanied by rising inflation?"


A week or two ago,  
WideRight : 5/26/2015 4:44 pm : link
Yellen made a public statement about "asset classes attaining speculative valuations" or something like that.

She was completely ignored.

Years ago, Greespan would say something like and the market would drop 1-2% in a day.

Fed has a credibility issue. Asset bubbles are considered part of the witches brew.
maybe its more about that the many past bubble/bubbles  
idiotsavant : 5/26/2015 5:09 pm : link
created distortions that never went away, fundamentally changed the nature of the entire economy, which changes survived the 'recovery' period, and which we take for granted, or that you guys don't see as distortions because you work in the economy and or invest in it closely, which is complex and very engaging.

maybe its not as much about the short term (2 to 3 years) bubbles and etc. pricing, the market and fluff/air or not, but more about fundamentals.

maybe a interest rate increase will help address the fundamentals, maybe not, how the heck would I know, acting in other ways, making the economy less about the markets and investing broadly, and more about other activities.

in other words, if the market has been divorced from the real economy, how will putting dampers on the market, or not, help, or not the real economy?

not to crap on all the good it did for 30 years or whatever it was, not at all, the cheap money changed the entire world and in many great, mind boggling, and lasting ways,...but maybe its time for a slight change.
the cheap money and the lowering  
idiotsavant : 5/26/2015 5:16 pm : link
from the Carter era high tax rates? similar in effect? allowing people to invest / money to be invested which changed the world radically and in amazing ways?

Just suggesting, (and I have no idea and am not in favor of tax increases) that that horse ran out of room to run after 30 years.
The system is rigged, It's a big con  
eli10 : 5/26/2015 5:48 pm : link
I stopped putting my money in 401ks etc. I take that money and put down extra on my mortgage principle every month. Much better,
Is it cheap?  
WideRight : 5/26/2015 5:50 pm : link
relative to others (that matter), our is getting more expensive.


Rates - ( New Window )
.  
Bill2 : 5/26/2015 5:51 pm : link
amazing
.  
Bill2 : 5/26/2015 5:53 pm : link
Good points WideRight
brazil labor participation rate  
idiotsavant : 5/26/2015 6:10 pm : link



which. I have really no idea here, darts in the dark, but Brazil has very high prime rate? and has had? (inflation also I guess)
Two things I`ve never understood.  
TJ : 5/26/2015 6:11 pm : link
1) Low interest rates should be bad for bond sales. But it appears people are willing to buy US debt paper just as fast as it can be printed. Apparently there is no bond rate so low that there won't be a large pool of investors looking to avoid all risk.
Is that true?

2) It's my understanding that Obama's wall street "reforms" are strictly cosmetic and that nothing at all has been done to either punish those who caused the recent crash or to prevent it from happening again tomorrow.
So why is anyone happy about investing in the market?
international labor participation rates  
idiotsavant : 5/26/2015 6:14 pm : link



Bond purchases are based on what you expect to get (average return)  
kicker : 5/26/2015 6:16 pm : link
and the volatility in these returns, all adjusted for risk.

But, since people are risk averse, they are willing to lose investment income to protect the downside. Bond purchase, in part, reflect this aversion.
india, brazil, even mexico to some extent  
idiotsavant : 5/26/2015 6:19 pm : link
ok, we don't want to be third worlders, however, all relatively high prime rates and higher labor participation rates than we have.

of course, I don't have any idea here if there is a correlation other than people live in poverty and get paid low wages in those places, which obviously sucks, but...is there another correlation.
reasons to devalue I guess  
idiotsavant : 5/26/2015 6:29 pm : link
.
Keep in mind labor participation rates...  
Dunedin81 : 5/26/2015 7:18 pm : link
are dependent on a number of factors, including median age, social norms regarding a woman's role in the home and in society, etc etc. So some of those differences can be explained on those grounds.
brazil  
giantfan2000 : 5/26/2015 7:28 pm : link
the minimum wage in brazil is $310 a month
manh  
Bill2 : 5/26/2015 8:30 pm : link
have to think about it but maybe all the misplaced faith in applied math rigor( when the drivers of analytical outcomes are the assumptions and not the calculative precision)has allowed risk reward pursuits in asset classes...while the same kind of math smooths out distribution channel signals that used to cause pricing and inventory dislocations?

Or the measure of inflation is for the birds and no one has pricing power anymore for their are so few barriers to entry when rates are so low for so long?

Have to think about it.

But we do know that Summers and Greenspan wrote about this kind of policy after the 87 crash and we as a nation followed that playbook ever since Bush I.
A model based upon accelerating technological change (ATC)....  
manh george : 5/26/2015 8:42 pm : link
explains a lot of what is happening, including Yellin's inability to change any of it. Let's just stay in the US, for now:

1) Labor's share of total national income has been shrinking for reasons starting with globalization and expanded by ATC. Owner's share has been increasing, sharply. That is consistent with high equity valuations.

2) ATC has been and will continue to put downward pressure on price changes. The energy story in the US is partly technological.

3) Inflation just isn't going to grow, if the technologists/futurists are at all correct. So, need for Fed tightening will be extremely limited, and the glide-path to higher short-term rates will be exceedingly slow.

4) This also feeds back to lower long-term rates while giving debt investors such as pension funds and insurance companies nowhere to go to get higher rates. It also feeds higher equity valuations, because the after-tax cost of borrowing cost of is low enough to incent companies to borrow in stockholder-friendly ways (e.g., buybacks).

4) Inflation is already mis-measured on the upside, because productivity is mis-measured on the downside.

Don't believe #4? OK, where is the ubiquitous availability of low-cost cell phones in the productivity numbers? (It isn't.)

Martin Ford, in his depressingly excellent book "The Rise of the Robots" gives another example. A company specializing in complex calculations estimated that a given calculation would have taken 260 years on a single computer. Now, it goes to a cloud service provided by Amazon, and uses tens of thousands of massively parallel computers via the cloud, to do the entire calculation in 18 hours. Where is that in the productivity numbers? (It isn't.)

Moore's law, which has at least 10 more years to go before shifting to new technologies (which will exist in time), AI, machine learning, deep learning, big data, software that is improving vastly faster than Moore's law, collapsing prices on sensors, robotics, and on and on and on are going to eat away at jobs, while also giving bigger shares to winners/owners, and keeping massive downward pressure on the inflation rate. This model explains an awful lot, including high-priced stocks and low interest rates, neither of which Yellin can change--even though most economists don't believe it yet, because they can't measure it very well yet.

How we as a society handle this, I have no idea, and it scares the shit out of me. I just believe strongly from everything I read and the technologist/futurists I know that it's happening. There are upsides, like exploding longevity and collapsing deaths from most major diseases. However, the downsides seem much worse, for at least the next 10-20 years.

This that all of the things ATC explains are from correlation, not causation? Fine, but keep up with the changes that are happening.

On Ford's book, in the NYT book review:
Quote:
“As Martin Ford documents in Rise of the Robots, the job-eating maw of technology now threatens even the nimblest and most expensively educated...the human consequences of robotization are already upon us, and skillfully chronicled here.


It isn't really just Robotization, btw. It's automation and AI writ large. The share of US jobs in manufacturing is already down from 30% in 1950 to about 8% now.

The Amazon page is linked.


Link - ( New Window )
Reb, to partially answer your initial not-so-dumb question...  
manh george : 5/26/2015 8:46 pm : link
when valuations are high, they also tend to be inherently volatile, and subject to high sensitivity to external forces such as the Fed outlook. Doesn't mean that outlook is correct, just that it causes large feedback loops in asset price valuation, along with lots of other factors.
Bill, to partly answer your question...  
manh george : 5/26/2015 8:52 pm : link
Ford notes that the concerns about ATC have been around at least since the 70's. What is different this time as I understand it, was described in another futurist book I read most of: "Our Digital Destiny."

As the author notes, what appears to be different this time is that we are now undergoing a transition from an analog world with digital inputs, to a digital world. As others have described it, we may now be seeing digitization unfold as the first true general purpose technology since electricity. And, like electricity, it doesn't change things much...until it does. Electricity took about 50 years. Things move a lot faster now.
.  
Bill2 : 5/26/2015 9:29 pm : link
Manh, with all due respect I am slow to accept causation for as much of all the phenomenon we see ( although math underlying risk instruments and forecasting is a form of technology). Same claims from Durkheim after touring the River Rouge plant with Henry Ford. Ditto Daniel Bell. Ditto C Wright Mills. Many of the new technologies do not impact all lives like rail, steam, gas, refrigeration, electricity do. No doubt information explosions impact how we few live and get factoids but making a white shirt is making a white shirt the world over. All in all I am not ready to make a conclusion that I am comfortable with just yet. Still digesting a lot of different things I see and hear and read.
I wouldn't expect otherwise.  
manh george : 5/26/2015 9:43 pm : link
But when robots can:

1) Learn to make white shirts; and

2) Transfer that primitive learning to 10,000 other robots...

then the world has truly changed--in my view.

I have a co-worker comes out of the biotech AI field, and now runs a trading department while also designing the technology for our entire division. No dummy, he. He has been my mentor on all of this, and I wouldn't be nearly as confident about the correlation part without his teachings.

I strongly encourage the Martin Ford book.
.  
Bill2 : 5/26/2015 10:59 pm : link
I'd be careful of textile industry examples of disaggregation via robots. I was close to those attempts with Japan and later India. The labor cost is so low right now. And political stability fights back against a lot of these claims.

To be honest no industry is more vulnerable to techno obsolescence than FIRE. Even there it is decades to work out the last 5%. Sort of like IBMs 25 year quest to beat the chess champion. Easy to get to beating masters. Much longer to beat some grand masters. Eventually took teams and generations of " machine" to beat a champion. There is one heck of a lot of reasons besides technology behind the erosion of our manufacturing base.

But mainly I think the trend will be less than its promulgators claim because other completely different trends on the move will exert more influence on human affairs sooner and more completely. Imho
Yes, it took a long time to beat Kasparov.  
manh george : 5/26/2015 11:48 pm : link
And now your cell phone can do it. The triumph in Jeopardy was more impressive, and faster. And has more applicability to the real world.


Where we go from here...well, we will see, pretty soon, I think.
of course, labor participation rates  
idiotsavant : 5/27/2015 8:29 am : link
in societies where the price for goods is so darn low, people who make them are half starved anyway, and there are no women in the workplace, the rates for men are higher.

But, being a total naif on econ,

Question? are there any other mechanisms? ones to slightly lower the dollar and marginally help us build our own manufacturing sector?

keep in mind, it is not all about exports, Brazil, with the huge high rates (13%?) and very high labor participation rates, some of what they manufacture is obviously sold at a low enough price right there in country.

.brazil  
idiotsavant : 5/27/2015 9:28 am : link
comparative debt to GDP of countries listed in the rates link above  
idiotsavant : 5/27/2015 9:54 am : link
regarding separation of the GDP from the wage economy  
idiotsavant : 5/27/2015 10:28 am : link
last one  
idiotsavant : 5/27/2015 10:35 am : link
And OMG  
WideRight : 5/27/2015 11:07 am : link
The Dow is up triple digits today.

idiotsavant...  
Dan in the Springs : 5/27/2015 2:03 pm : link
those are very interesting charts. I like how you leave the narrative out of it - let each develop their own for the data.

The last chart along with what Manh George has been describing here has me preaching to my students - economic times are changing. Get in the investor class. You have no chance if you live paycheck to paycheck.

In response to the O.P., I have experienced many of the same feelings from time to time. I remember good economic news battering my investment portfolio as it foretold of changes by the fed, and being really upset as it seemed to force me to root for bad info.

I understand that the market is all speculation, and as such is driven by the rumor, not the news, etc.

Nowadays it's not even driven by the rumor or the news so much as it is by the algorithms/pricing models. I can't do that kind of math so I've given up trying to explain the day's actions on the marketplace, and even when I hear people who seem to explain yesterday's results in a rational way I don't invest on their recommendation. One has to look at the big picture, period.
"Battering My Investment Portfolio"  
baadbill : 5/27/2015 8:14 pm : link
I get the sense that not many on this thread have done much studying of finance or the theory of investing. Except for those who are already retired and no longer contributing to their savings - everyone else should be praying for - and CHEERING - having your investment portfolio battered. The best thing that can happen to you if you are making regular contributions to your portfolio, is for stocks to go down every day further and further until the day you retire - and then for stocks to go up every day that you are retired. Of course, that's never happened and not going to happen.

But - the LAST thing anyone should want is for stocks to go UP. You want stocks to go DOWN, not UP (again, unless you are retired and no longer a contributor, but a spender of your portfolio).

So, CHEER when the market goes down and you are buying more shares with your weekly contribution. And Pray that next week stocks get cheaper still, so you buy even MORE shares. And my goodness, if you can possibly be fortunate enough to be contributing when there is an actual CRASH (say a 40-50% drop), borrow money on your house, beg borrow and steal, do whatever you can to BUY BUY BUY cheap stocks. And pray for it to go down even further.

Investing is very counter intuitive. But it is damn important to your future. If you don't understand the reasoning behind anything I've said above, then you owe it to yourself to read a few of the basic investing books recommended by most universities (Chicago School of Economics is probably the best and I'm sure they have a great finance section with recommended books. Random Walk Down Wallstreet is a great starter).

But - remember - you should be pissed off when stock prices rise because your weekly contribution is now purchasing fewer shares. And you should be pleased as punch when prices fall, because your contribution is purchasing more shares (and remember - the GOAL is to accumulate the most SHARES, not dollar value - the dollar value will come when the price of the shares eventually rises - which it will do over a 40 year investment period - but meanwhile, you want to buy LOTS of shares, and the lower the stock market goes, the more shares you will be buying).
You should keep  
Rob in CT/NYC : 5/27/2015 8:34 pm : link
Your money in a mattress - holy my fuck.
RE: You should keep  
baadbill : 5/27/2015 8:40 pm : link
In comment 12303352 Rob in CT/NYC said:
Quote:
Your money in a mattress - holy my fuck.


If you are talking to me, then you are, sadly, an idiot and do not have a clue what you are talking about.
...  
kicker : 5/27/2015 8:44 pm : link
Wow.
Indeed I was talking about you  
Rob in CT/NYC : 5/27/2015 8:44 pm : link
You should never invest - go to the track. Spend it on hookers and blow. I don't even know where to start to show you where you are wrong.
By the way, in no way, shape, or form does the Chicago School  
kicker : 5/27/2015 8:47 pm : link
endorse rooting for the stock market to go lower, lower, lower. That goes against their whole notion of rational expectations...
RE: Indeed I was talking about you  
baadbill : 5/27/2015 8:52 pm : link
In comment 12303370 Rob in CT/NYC said:
Quote:
You should never invest - go to the track. Spend it on hookers and blow. I don't even know where to start to show you where you are wrong.


Well, let's see. Try reading John Bogle. Warren Buffett. Do you know who they are?

Do you have a clue how to project future returns? Which is better for future returns, a current stock market with high prices (let's say an overall PE of 30) or a current stock market with very low prices (say an overall PE of 7)? Do you know the answer? Do you know WHY the answer is what it is? My guess is you don't have a clue.

I suspect you also think that owning individual stocks is a wise thing to do. Or actively managed mutual funds? That somehow you (and others) can beat a passive index fund, right?

I can only laugh at you. But please don't spread your garbage to others at BBI (or elsewhere in the world) whose financial futures can be ruined by someone who hasn't studied finance like yourself.
Tell us, oh wise one, how expectations theory  
kicker : 5/27/2015 8:57 pm : link
does not play a role in the demise of this intelligent investing strategy you have laid out?

You know, the one espoused by the Chicago School of Economics.
if you could stop ranting, maybe I could help you  
Rob in CT/NYC : 5/27/2015 8:58 pm : link
How about we start with this notion that a best case scenario is that the market goes down every day until you retire? Now, you say it won't ever happen (agreed), but if I show you how wrong that is, will you try and acknowledge that maybe reading a book isn't the ultimate achievement in investing? Agreed?
baadbill  
manh george : 5/27/2015 9:04 pm : link
Last time I looked, Rob was a respected Wall Street research analyst. (I haven't looked in a while. He may be running a hamburger joint for all I know, but the analytical skills/market knowledge don't disappear.)

You?
RE: By the way, in no way, shape, or form does the Chicago School  
baadbill : 5/27/2015 9:05 pm : link
In comment 12303374 kicker said:
Quote:
endorse rooting for the stock market to go lower, lower, lower. That goes against their whole notion of rational expectations...


CRISP doesn't "root" for stock prices to do anything. But if you have a clue about investing - you'd understand why every professor at CRISP would teach you that during the accumulation phase (i.e. making regular additional investments), if you are lucky enough for stock prices to continually go DOWN during your accumulation period, you will end up accumulating more shares and will end up wealthier at the end of the period.

Take two theoretical investors, A and B. Both invest in the S&P500 for 40 years. Both start with the S&P500 at 12,000 and end with the S&P500 at 36,000 when they are 65.

Investor A sees the S&P500 go DOWN continually over 40 years from 12,000 down to 1,000 when he is 64. Then in the last 12 months it goes on a huge boom and reaches 36,000 as promised.

Investor B sees the S&P go UP continually over 40 years so that it reaches 100,000 at age 64. Then in the last 12 months it goes on a huge downward spiral and ends at 36,000 as promised.

Which investor has the better performing portfolio at 65? Investor A who saw stocks go down during his entire period of accumulation? Or Investor B who stocks go UP during his entire period of accumulation?

The answer is quite simple. Obviously the example I've provided is exaggerated for purposes of illustration. The point is, that you should be investing in an index fund and accumulating (acquiring) as many SHARES as possible (which means low price - the lower the better) - with the knowledge that reversion to the mean requires that the prices will go up in the later stages of acquisition, making you much wealthier than someone who sees the higher prices early, and reversion to the mean results in fewer shares acquired and lower price performance at the end.

RE: baadbill  
baadbill : 5/27/2015 9:06 pm : link
In comment 12303410 manh george said:
Quote:
Last time I looked, Rob was a respected Wall Street research analyst. (I haven't looked in a while. He may be running a hamburger joint for all I know, but the analytical skills/market knowledge don't disappear.)

You?


Don't make me laugh. John Bogle and Warren Buffett will tell you there is no such thing. Wall Street is nothing but a snow job - they are salespeople.
That is such an awful example of what CRISP professors  
kicker : 5/27/2015 9:09 pm : link
actually do talk about.

Again, the school that was made famous for "rational expectations", and not once do you actually put that into your silly tripe?

How, perchance, do risk strategies play out for a risk averse and risk loving individual if the expectation is that stocks will, in general, decline. And how does this differ from an individual who expects that stocks will rise?

How do these expectations play a role in shaping the broader market, as you are a market taker? Do these expectations drive down stock prices even when fundamentals say they should increase, and how does this affect returns?

But, we've got it. You have read like 3 books. Enlightening.
By the way, I'm pretty sure everyone  
kicker : 5/27/2015 9:12 pm : link
would take the word of someone who has worked in (or near the periphery of) the industry as an analyst over someone who's sole claim to fame is reading. Bravo.
RE: That is such an awful example of what CRISP professors  
baadbill : 5/27/2015 9:12 pm : link
In comment 12303423 kicker said:
Quote:
actually do talk about.

Again, the school that was made famous for "rational expectations", and not once do you actually put that into your silly tripe?

How, perchance, do risk strategies play out for a risk averse and risk loving individual if the expectation is that stocks will, in general, decline. And how does this differ from an individual who expects that stocks will rise?

How do these expectations play a role in shaping the broader market, as you are a market taker? Do these expectations drive down stock prices even when fundamentals say they should increase, and how does this affect returns?

But, we've got it. You have read like 3 books. Enlightening.


Try explaining the example I gave above re Investor A and B. You do understand that Investor A is the winner by a large margin, right? Do you understand why? And the consequences of why?
Well, I spend my days doing market strategy on WS.  
manh george : 5/27/2015 9:13 pm : link
II ranked over 90% of the time since they started covering my sector, and I don't do ANY selling in my work, I can assure you.

There are Street analysts all over the place with fabulous knowledge about their area of expertise. And yes, lots of salesmen. Smart institutional clients know how to tell the difference.

So, all you have done is highlighted how much you don't know about the profession, or the investment process.

Thanks for playing.
Please tell me how the fuck your simple example  
kicker : 5/27/2015 9:14 pm : link
is even close to realistic, why a contrived example to arrive at a pre-determined answer should be given any weight (what happens if it doesn't rise, and continues to fall), and how you can ignore the fact expectations (again, CRISP) factor into future values of stock prices?

Change some numbers (since, you know, there are an infinite number of permutations to your example being incorrect) and your point goes bye-bye.

Especially since you have to rely on the wonderful timing of the market and your retirement...
Baadbill  
manh george : 5/27/2015 9:15 pm : link
Yes, under your hypothetical, you are correct. The only leeeeetle problem is that your hypothetical looks nothing like the way markets behave.

Nice try, though.
RE: By the way, I'm pretty sure everyone  
baadbill : 5/27/2015 9:16 pm : link
In comment 12303429 kicker said:
Quote:
would take the word of someone who has worked in (or near the periphery of) the industry as an analyst over someone who's sole claim to fame is reading. Bravo.


Again, if you like sales people, then be my guest. Go to some stock broker and lose your shirt. Then try some annuities from an insurance company.

On the other hand, maybe you should try reading some books that come out of CRISP and maybe you just MIGHT save your financial future. No skin off my back. I'll stick with Warren Buffett and John Bogle. Peter Lynch. Shiller from Yale. Fama from CRISP.
RE: RE: By the way, I'm pretty sure everyone  
kicker : 5/27/2015 9:18 pm : link
In comment 12303440 baadbill said:
Quote:
In comment 12303429 kicker said:


Quote:


would take the word of someone who has worked in (or near the periphery of) the industry as an analyst over someone who's sole claim to fame is reading. Bravo.



Again, if you like sales people, then be my guest. Go to some stock broker and lose your shirt. Then try some annuities from an insurance company.

On the other hand, maybe you should try reading some books that come out of CRISP and maybe you just MIGHT save your financial future. No skin off my back. I'll stick with Warren Buffett and John Bogle. Peter Lynch. Shiller from Yale. Fama from CRISP.


If you've read Fama, then please, incorporate expectations into your theory. I've asked, and you haven't answered.

By the way, I've actually taken courses from Fama. This is not what this theory says. At all.
You have a very simple (and incorrect) view  
kicker : 5/27/2015 9:18 pm : link
of the world.

RE: RE: By the way, in no way, shape, or form does the Chicago School  
Rob in CT/NYC : 5/27/2015 9:18 pm : link
In comment 12303413 baadbill said:
Quote:
In comment 12303374 kicker said:


Quote:


endorse rooting for the stock market to go lower, lower, lower. That goes against their whole notion of rational expectations...



CRISP doesn't "root" for stock prices to do anything. But if you have a clue about investing - you'd understand why every professor at CRISP would teach you that during the accumulation phase (i.e. making regular additional investments), if you are lucky enough for stock prices to continually go DOWN during your accumulation period, you will end up accumulating more shares and will end up wealthier at the end of the period.

Take two theoretical investors, A and B. Both invest in the S&P500 for 40 years. Both start with the S&P500 at 12,000 and end with the S&P500 at 36,000 when they are 65.

Investor A sees the S&P500 go DOWN continually over 40 years from 12,000 down to 1,000 when he is 64. Then in the last 12 months it goes on a huge boom and reaches 36,000 as promised.

Investor B sees the S&P go UP continually over 40 years so that it reaches 100,000 at age 64. Then in the last 12 months it goes on a huge downward spiral and ends at 36,000 as promised.

Which investor has the better performing portfolio at 65? Investor A who saw stocks go down during his entire period of accumulation? Or Investor B who stocks go UP during his entire period of accumulation?

The answer is quite simple. Obviously the example I've provided is exaggerated for purposes of illustration. The point is, that you should be investing in an index fund and accumulating (acquiring) as many SHARES as possible (which means low price - the lower the better) - with the knowledge that reversion to the mean requires that the prices will go up in the later stages of acquisition, making you much wealthier than someone who sees the higher prices early, and reversion to the mean results in fewer shares acquired and lower price performance at the end.


Great example...so long as the market is up 36x in the final year...let me make up one...market goes down 5% per year for one investor, up 5% for another, 40 year contribution period, $1,000 per year.

Down 5% investor wins on your all-important number of shares metric, enters distribution phase with $17,000 and change.

Up 5% - a lot less shares, but $121,000 in retirement savings.

To be clear, you want the market to go down until retirement?
1 question quiz for Baadbill.  
manh george : 5/27/2015 9:20 pm : link
Do you know this difference between a stockbroker and an institutional-LEVEL investment strategist or research analyst?

The evidence is that you do not.

Added questions for Baadbill.  
manh george : 5/27/2015 9:23 pm : link
1) If the market acts as you suggest for 40 years, what kind of shape will the economy be in?

2) Given that, would you have a job, let alone be able to retire?
And just as an FYI  
Rob in CT/NYC : 5/27/2015 9:24 pm : link
One person you gave your brilliant advice to, likely retired the year BEFORE the market went up 36x. How did that work out for him?
RE: Baadbill  
baadbill : 5/27/2015 9:25 pm : link
In comment 12303438 manh george said:
Quote:
Yes, under your hypothetical, you are correct. The only leeeeetle problem is that your hypothetical looks nothing like the way markets behave.

Nice try, though.


That isn't the point. Of course it doesn't represent the way markets behave. It simply makes the point that during one's accumulation phase, LOWER prices early in accumulation mean two things:
1. You will acquire more shares with the same money invested as does someone unlucky enough to be experiencing HIGHER prices early in their accumulation phase.
2. Reversion to the mean dictates that your lower prices will, over time, revert to higher prices, hopefully by the time you retire. On the other hand, someone who starts with HIGH prices not only acquires fewer shares, but reversion to the mean dictates that over time prices will fall.

Of course there are likely to be mutliple market cycles during an investing lifetime, with a generally upward bias.

But the point is that there is nothing better than a crash during one's accumulation phase. It is an opportunity to buy MORE SHARES at much cheaper prices (think of them as CARS - but now instead of $30,000 per car you're paying $5,000 for the same car).

I'm only talking about what one would wish for (root for) - but it is out of our control and we are subject to lady luck. Just hope that when you retire, the day before you actually retire is at a 30 year market low, which will virtually guarantee RISING stock prices during the early part of your retirement which is exactly what you need.

Likewise, if you could control it, you would LIKE to have low prices as often as possible during your accumulation phase. It is counter intuitive until someone explains it or you actually crunch the numbers. Bottom line - BUY LOW. And if possible - BUY EVEN LOWER.


By the way. The fact that the only way your investment  
kicker : 5/27/2015 9:26 pm : link
example can hold water is with a "reversion to the mean" appeal is pretty appalling. Because it's incorrect.

You do understand what it entails, right? That, because stock prices are falling each period, consistently, the average stock price will be very low as well?

Buy low is different from  
kicker : 5/27/2015 9:27 pm : link
hope for luck and the stock market to tank...

Not even close.
So, essentially, if you live in a perfect world with perfect  
kicker : 5/27/2015 9:28 pm : link
foresight, bill's plan is the way to go.

If not, run the fuck away from it as fast as you can.
What happens if the move lower  
Rob in CT/NYC : 5/27/2015 9:28 pm : link
is actually the mean reverting move?
Btw...  
manh george : 5/27/2015 9:30 pm : link
Buffet believes more in investment strategists and analysts than he does in trying to time the market.
By the way, may want to do a bit more reading. Greg  
kicker : 5/27/2015 9:33 pm : link
Samsa at Duke has found the following two things that are true for mean reverting stocks. Neither of which are supported by your example:

1. Low volatility in prices (isn't happening)
2. Low business risk (isn't happening).

Agreed with everything Bill posted.  
BrettNYG10 : 5/27/2015 9:34 pm : link
Great work.
The worst thing that can be happening right now is a market  
baadbill : 5/27/2015 9:34 pm : link
with a Shiller PE of 27.

The only people benefit from such a market are those that retired over the past 7 years and kept a fair percentage of their portfolio properly invested in stocks.

Anyone accumulating assets right now are buying fewer shares and paying a very high price (nevertheless, for someone young enough, better to be steadily acquiring stocks at any price that not investing at all) - but this is not a good market for anyone in the accumulating phase. What they should want is a SHILLER PE of 7.

We will see a PE of 7 again. And people who are lucky enough to be in their accumulation phase when the market is at a Shiller PE of 7 will be very wealthy some day - much much wealthier than those who are accumulating shares at today's prices of a SHILLER PE of 27.

This isn't an investment theory. This isn't an investment philosophy. This has to do with reversion to the mean. This has to do with the fact that the markets have cycles. This has to do with understanding that you aren't MAKING money just because the market goes up - you only make that money if you sell (which is a different story - but trading or trying to time a market is generally a very risky behavior).
...  
kicker : 5/27/2015 9:35 pm : link
You don't understand reversion to the mean, and how there is zero support for it in your examples, and you expect us to swallow?
RE: So, essentially, if you live in a perfect world with perfect  
baadbill : 5/27/2015 9:36 pm : link
In comment 12303467 kicker said:
Quote:
foresight, bill's plan is the way to go.

If not, run the fuck away from it as fast as you can.


I have not talked about a PLAN of any kind. I made a simple statement. Lower prices are a good thing for someone BUYING stocks. If you don't understand that, then I can't help you.
RE: Btw...  
baadbill : 5/27/2015 9:37 pm : link
In comment 12303471 manh george said:
Quote:
Buffet believes more in investment strategists and analysts than he does in trying to time the market.


Trying to time the market is plain stupid.
By the way, what about the finding that mean  
kicker : 5/27/2015 9:38 pm : link
reversion in stock prices is positively correlated to stock returns?

Or, as markets continuously fall, the probability of mean reversion gets weaker.
RE: RE: So, essentially, if you live in a perfect world with perfect  
kicker : 5/27/2015 9:39 pm : link
In comment 12303482 baadbill said:
Quote:
In comment 12303467 kicker said:


Quote:


foresight, bill's plan is the way to go.

If not, run the fuck away from it as fast as you can.



I have not talked about a PLAN of any kind. I made a simple statement. Lower prices are a good thing for someone BUYING stocks. If you don't understand that, then I can't help you.


Oh, trust me, I don't want to follow your path to success...

By the way, how do you not know what a plan is? Because you perfectly summed one up in your doozy of a post.
I'll try to say it one more time.  
baadbill : 5/27/2015 9:44 pm : link
I am not talking about an investment plan. I am not talking about how markets work. I am not talking about investing philosophy. I am not talking about what you should or shouldn't do. I am not talking about what stocks do or don't do.

I am making a very simple statement. To the extent that you are fortunate enough to be able to experience very low stock prices, be happy about that and try to find a way to invest even more money than normal into stocks when they are so low.

When stocks are booming - be wary. It is foolish to try to time the market (i.e. sell) but it certainly isn't a time to be happy (and it is natural for us to be feeling good about our investments when they double in price - it makes us feel as though our strategy is working - and it is - but the reality is that now that prices have doubled, we are now acquiring 50% fewer shares with each weekly contribution than we did 10 years earlier before stocks doubled).

So, in general times, so long as I am in my accumulation phase, I WANT lower prices.
...  
kicker : 5/27/2015 9:46 pm : link
Yeah, you may want to re-read your doozy of an opening post.
RE: ...  
baadbill : 5/27/2015 9:51 pm : link
In comment 12303502 kicker said:
Quote:
Yeah, you may want to re-read your doozy of an opening post.


Just read my doozy initial post. All I said is that investors in their accumulation phase should be HAPPY to pay lower prices for stocks (buy low) and be looking for higher prices only after they've retired (sell high).

Not exactly a difficult concept to follow. But one that most investors don't get. They get all excited by a rising market (like the one we have now) when in fact they are still years away from retirement and simply OVER PAYING for stocks with their weekly contributions. They would be so much more better off with LOWER stock prices, not HIGHER.

This isn't a philosphy. This isn't a PLAN. But somehow it is difficult for people to wrap their brains around.
What's even more astounding is your selective amnesia as  
kicker : 5/27/2015 9:52 pm : link
to what your post actually said.
RE:  
Dan in the Springs : 5/28/2015 12:02 am : link
In comment 12303314 baadbill said:
Quote:
I get the sense that not many on this thread have done much studying of finance or the theory of investing. Except for those who are already retired and no longer contributing to their savings - everyone else should be praying for - and CHEERING - having your investment portfolio battered. The best thing that can happen to you if you are making regular contributions to your portfolio, is for stocks to go down every day further and further until the day you retire - and then for stocks to go up every day that you are retired. Of course, that's never happened and not going to happen.

But - the LAST thing anyone should want is for stocks to go UP. You want stocks to go DOWN, not UP (again, unless you are retired and no longer a contributor, but a spender of your portfolio).

So, CHEER when the market goes down and you are buying more shares with your weekly contribution. And Pray that next week stocks get cheaper still, so you buy even MORE shares. And my goodness, if you can possibly be fortunate enough to be contributing when there is an actual CRASH (say a 40-50% drop), borrow money on your house, beg borrow and steal, do whatever you can to BUY BUY BUY cheap stocks. And pray for it to go down even further.

Investing is very counter intuitive. But it is damn important to your future. If you don't understand the reasoning behind anything I've said above, then you owe it to yourself to read a few of the basic investing books recommended by most universities (Chicago School of Economics is probably the best and I'm sure they have a great finance section with recommended books. Random Walk Down Wallstreet is a great starter).

But - remember - you should be pissed off when stock prices rise because your weekly contribution is now purchasing fewer shares. And you should be pleased as punch when prices fall, because your contribution is purchasing more shares (and remember - the GOAL is to accumulate the most SHARES, not dollar value - the dollar value will come when the price of the shares eventually rises - which it will do over a 40 year investment period - but meanwhile, you want to buy LOTS of shares, and the lower the stock market goes, the more shares you will be buying).


baadbill - a lot of assumptions in your post here. You might assume that all investment portfolios are built the same way, but you'd be wrong about that. What I've managed to accumulate was built over time, and included weekly investment contributions. But that's not where the bulk of my portfolio came from.

Have you ever heard of stock options? I worked for a firm that payed a sizeable annual bonus, regularly the size of my annual salary. It was paid in stock options with a three-year vesting schedule. I didn't get to control that portion of my investment portfolio, but if you do the math you can see that roughly three times my annual salary was invested in a single company - the firm that I worked for.

Now I know that no advisor would encourage someone to take such a large position on a single company, but I don't know anyone turning that money down either.

You know what really sucks? When your shares vest right after positive economic news that drops your share price overnight.

I sure as heck was not rooting for my firm's price to go down.

You're right that I don't know much about investing. Let's put it this way - I know enough about investing to know I can't give advice on investing and/or lecture others on their investments.
fwiw baadbill...  
Dan in the Springs : 5/28/2015 4:18 am : link
Your point (that it is better to buy low and sell high) is correct.

On the surface what you say isn't that bad - that in theory one should desire lower prices during purchase periods and prices to rise during selling periods.

From reading through this thread I see two reasons for the very negative reactions to your post.

First, the way you come across should embarrass you. Do you really think other posters on this thread know nothing about the market?

Second, you miss your mark with your statement. I understand the idea of looking for the silver lining to a market downturn - "hey, it's a great time to be buying!", as does everyone else on this thread. That's a far cry from how you came across, suggesting we should cheer market declines unless we are retired. Posters in the know here have tried to illuminate why to you.

Do yourself a favor and remove the impression you've got that you're talking to a bunch of simpletons. Work on finding more gracious ways to discuss the market with others. You'll find that you not only make more friends, but you may learn more about the market along the way.
fwiw baadbill...  
Dan in the Springs : 5/28/2015 4:21 am : link
Your point (that it is better to buy low and sell high) is correct.

On the surface what you say isn't that bad - that in theory one should desire lower prices during purchase periods and prices to rise during selling periods.

From reading through this thread I see two reasons for the very negative reactions to your post.

First, the way you come across should embarrass you. Do you really think other posters on this thread know nothing about the market?

Second, you miss your mark with your statement. I understand the idea of looking for the silver lining to a market downturn - "hey, it's a great time to be buying!", as does everyone else on this thread. That's a far cry from how you came across, suggesting we should cheer market declines unless we are retired. Posters in the know here have tried to illuminate why to you.

Do yourself a favor and remove the impression you've got that you're talking to a bunch of simpletons. Work on finding more gracious ways to discuss the market with others. You'll find that you not only make more friends, but you may learn more about the market along the way.
.  
Bill2 : 5/28/2015 7:28 am : link
Bill I don't mean to pile on but I submit that:

The thing currently called the market; isnt.

Warren Buffet no longer invests in the same market of information and opportunity that anymore than 3 people get to come their way. Much less access to the political cooperation required to risk adjust.

Market and investment analysts produce collateral for salespeople and sell wearing cloaks anyone can see through. Whose name is on the transaction record is secondary to who participated in the go to market strategy of institutions whose purpose is to generate transactions from the pool of possible funds.
.  
Bill2 : 5/28/2015 7:30 am : link
Bill I don't mean to pile on but I submit that:

The thing currently called the market; isnt.

Warren Buffet no longer invests in the same market of information and opportunity that anymore than 3 people get to come their way. Much less access to the political cooperation required to risk adjust.

Market and investment analysts produce collateral for salespeople and sell wearing cloaks anyone can see through. Whose name is on the transaction record is secondary to who participated in the go to market strategy of institutions whose purpose is to generate transactions from the pool of possible funds.

Nothing against selling or buying. It's a good thing. We need it. As long as no one is fooled as to what it is.
.  
Bill2 : 5/28/2015 7:31 am : link
Bill I don't mean to pile on but I submit that:

The thing currently called the market; isnt.

Warren Buffet no longer invests in the same market of information and opportunity that anymore than 3 people get to come their way. Much less access to the political cooperation required to risk adjust.

Market and investment analysts produce collateral for salespeople and sell wearing cloaks anyone can see through. Whose name is on the transaction record is secondary to who participated in the go to market strategy of institutions whose purpose is to generate transactions from the pool of possible funds.

Nothing against selling or buying. It's a good thing. We need it. As long as no one is fooled as to what it is.
.  
Bill2 : 5/28/2015 7:58 am : link
To be clear: what many read as " analysis" is nowhere close even if an argument and an analysis use the same statistical tools. The difference between brilliant argumentation and analysis is not in how the data is brilliantly tortured into confessing; the difference is in what question each seeks to answer and what alternative options each is prepared to consider along the way.

Argumentation is selling. the record of that ( a transaction) is just that...a record of who was assigned or who brought the emotion stirred by the argument into action
It was a little bit disheartening to see the back and forth on this  
idiotsavant : 5/28/2015 8:18 am : link
often it is just about semantics, one person or people get a little sloppy (often a new poster) about wording

...then our local experts jump all over his ass with the mis-attributed understandings of what he was trying to say

...then he comes back with more knowledge than they assumed...they get hackles up, having made an assumption about his ignorance that was a little bit excessive..and it is on.

Forgetting the specifics, but before any alleged hate speech (I missed it), GWG spoke about some stuff, regulars did not initially see that he had some bona-fides (agreeing or not) and the rest was a retrenchment from that initial imbroglio.
Fun stuff  
WideRight : 5/28/2015 8:21 am : link
Free football bulletin boards are the place for high quality investment advice.

Dan is right. Poor badbill was presumptuous and arogant, the very traits that get consistently humbled when playing the market. No point in arguing his hyptothetical, because its hypothetical. Lessons from '08 - '09 aren't likely to be be too valuable going foward, particularly since Fed policy favors high asset values. But badbill's contrarian element style is exactly what helps maintain a market. Every seller needs a buyer. Go for it Badbill!
putting that aside for a momment  
idiotsavant : 5/28/2015 8:24 am : link
I had been hoping that my betters would speak on the diminishment of the wages as % of GDP.

and on the relationship between monetary policy and national debt in light of what is the most important part for the nation work on and what to do next and why
Shut up with the fucking "alleged"  
Rob in CT/NYC : 5/28/2015 8:32 am : link
GWG spew was reproduced multiple times, and any consistent reading of his posts reinforced it. Pay attention.
OP was a dumb economics question  
WideRight : 5/28/2015 8:34 am : link
Might have to start a new thread for that one...

I would speculate there's a strong correlation between ATC and decreasing wages as a % GDP. But what it means going forward for a wage earner or investor is beyond me.
The Ghost of GWG  
WideRight : 5/28/2015 8:35 am : link
is still here?
ok, I tend to skip over the fights  
idiotsavant : 5/28/2015 8:36 am : link
its getting booring. I want to learn about econ.

check out the chart at about 10:35am yesterday and comment for me.

Stat!
thank you wide right  
idiotsavant : 5/28/2015 8:37 am : link
what is ATC, though?
One would assume that the effects of non-wage  
idiotsavant : 5/28/2015 8:39 am : link
components of GDP are much more 'clumpy', and thus act on the business cycle differently.
.  
idiotsavant : 5/28/2015 8:51 am : link
.  
Bill2 : 5/28/2015 8:52 am : link
Seriously and well intentioned:

Think about it. Just list what you know of the Brazil in the last decade. It's easy. That's the best way to learn. Don't assume monetary policy is at work. Then add that collection of variables. Everything after that is debate over snapshots from a still moving picture. Imho.

and as far as GWG. Forget the post you did not see. As if that means it did not exist. Forget who started what. ..does not matter...is not analyzable. Was his presence highly likely to allow debate or discussion or not? And what percent of the time and what percent of the posters.

A data point on one thread that you have what was a good discussion for you is not a trend or a pattern or a probability.

Imho. And I did not get into any disputes with him I remember. To me he was one of the few folks every year that wreck too many threads and are resistant to feedback. And seeking refuge in the idea that he was just contrarian is facile and incorrect. A wide range of views honorable discussed has been tolerated on the site for a long time.

Just food for thought

the problem with ' household disposable income' as a unit of measure  
idiotsavant : 5/28/2015 9:00 am : link
might be that it does not break out wages, which IMHO ought to be differentiated from social security and other benefits as well as from capital gains in order to form a more accurate image of what is going on:

''Household disposable income includes income from economic activity (wages and salaries; profits of self-employed business owners), property income (dividends, interests and rents), social benefits in cash (retirement pensions, unemployment benefits, family allowances, basic income support, etc.), and social transfers in kind (goods and services such as health care,, education and housing, received either free of charge or at reduced prices). Across the OECD,''

(OECD Better Life Index)
Bill2 thx and ok  
idiotsavant : 5/28/2015 9:03 am : link
. I am trying to break out wages generated disposable income from other sources of disposable income and looking for a chart.

don't have a clue, exploring
RE: thank you wide right  
WideRight : 5/28/2015 9:06 am : link
In comment 12303706 idiotsavant said:
Quote:
what is ATC, though?


Accelerating Technological Change

More economic value is being created by automated processes.

Frickin' bank machines can take 2-3 bucks on a transaction with no humans required, other than mainatainence. That stuff adds to GDP but not wages. And accelarate that.
add to WideRights information  
Bill2 : 5/28/2015 9:11 am : link
what industries has Brazil recently added to their mix? Are those human capital centric or transmission/raw resource centric?
this is not what I was looking for, but edifying  
idiotsavant : 5/28/2015 9:17 am : link
thank you guys  
idiotsavant : 5/28/2015 9:23 am : link
I get that now. I am trying to find data that breaks down the components of disposable income, and over time, (50 years?) because I think it will shed light.
eh  
Bill2 : 5/28/2015 9:37 am : link
In the Usa, 90% of all income above median income and below the top 2% is spent on taxes, insurances, health, home, transportation and food. Only 10% is disposable above the "choices" within the necessary buckets. So choice is available but true optionality beyond what a family comes to view as necessity is available to any but the top 2%. In general.
This has been true for decades and decades and good time and bad.

It's how a guy making 70 and a guy making $170 feel equally under pressure to just make their ends meet. Perception being reality.
btw  
Bill2 : 5/28/2015 9:40 am : link
Substitution is widely studied. But the data I indicated is a summation largely based on Runzhiemer.
in other words, two things:  
idiotsavant : 5/28/2015 9:46 am : link
1. Not breaking out the components of disposable income AND GDP has possibly clouded policy making.


2. There are probably other factors leading to the stagnation in wages as a part of GDP and DPI ..

...in -addition- to the technology thing you guys mentioned, and 'international wage/currency arbitrage' (is that a thing?) that people talk about.



Factors such as increasing cost of living makes the decision to work less rational,

factors to do with the relative benefits of a work/saving lifestyle as opposed to the invest entrepreneurial lifestyle.

OR something I don't understand to do with the monetary, as the charts imply, 'the chicken or the egg', it looks like inflation did not hurt us as much as we like to think back in the 'bad 1970's', since the wages grew slightly faster.
bill, sorry, i dont know the lingo  
idiotsavant : 5/28/2015 9:50 am : link
I am trying to look at -specific sources of income to individuals-

and break them out, then look at those in charts (charts that typically only show the whole unit (DPI) not broken down suchly)...and therefore perhaps which are not as enlightening.

I am not looking at how the DPI is split up AFTER it is earned, looking for sources of.

RE: in other words, two things:  
njm : 5/28/2015 9:50 am : link
In comment 12303843 idiotsavant said:
Quote:
it looks like inflation did not hurt us as much as we like to think back in the 'bad 1970's', since the wages grew slightly faster.


You're not taking into account the negative impact inflation had on home ownership and the tax impact of bracket creep.
In other words, how can we dismiss the 'sources of and  
idiotsavant : 5/28/2015 9:56 am : link
break-outs of what goes into income', without looking at the data? maybe disposable is the wrong measure, mea culpa then,

maybe I should go back to wages as % of GDP, but...


...if nobody can gain disposable from wages...


that is the entire problem in a nutshell.
NJM, I will take your word for it  
idiotsavant : 5/28/2015 10:00 am : link
but, maybe let's not be tied to housing and the market to such a degree that working people cannot feed back into the business cycle at all anymore.

as you can see in the chart, wage increases covered inflation, just enough to provide for them to do some feedback of their own into the business cycle, and however small per household, it was widespread, not clumpy.
RE: RE: in other words, two things:  
Dunedin81 : 5/28/2015 10:03 am : link
In comment 12303858 njm said:
Quote:
In comment 12303843 idiotsavant said:


Quote:


it looks like inflation did not hurt us as much as we like to think back in the 'bad 1970's', since the wages grew slightly faster.



You're not taking into account the negative impact inflation had on home ownership and the tax impact of bracket creep.


And also the fact that these increases in wages are generally quite uneven, as are the other ancillary "benefits" such as increased home prices (your home doubles in nominal price even as your fixed-rate mortgage payment holds steady). So what is a boon for some may be devastating for others.
of course, total newbie on econ  
idiotsavant : 5/28/2015 10:08 am : link
but, it may be telling that a google search and images search finds so little detail regarding break downs of what goes into GDP and DPI.

and, if I understood bill2 correctly, wage people have not had disposable in forever, so that is an obvious problem.

how can the nation make policy if it looks at big, lumpy, contradictory units of measure such as GDP and DPI without looking at the very divergent parts of them?

I am suggesting that 'income' ought not be all lumped as equal, it is not.
You're going in circles  
WideRight : 5/28/2015 10:45 am : link
You are correct that the lack of info the breakdown of disposable is telling. Its probably tells you income from different sources is the same, economically. Cash is cash.

But the fact that true disposable- discretionary- income has been steady and scarce for decades is not bad at all. People spend it. And the more things there are to spend it on, the better. It enriches their lives and drives the ecomony. 2/3 of GDP is consumer-driven. It also motivates people to earn more and be more productive. We are the most productive society in the world, as well as the biggest consumers. A materialistic heaven!
RE: It was a little bit disheartening to see the back and forth on this  
River Mike : 5/28/2015 10:50 am : link
In comment 12303676 idiotsavant said:
Quote:
often it is just about semantics, one person or people get a little sloppy (often a new poster) about wording

...then our local experts jump all over his ass with the mis-attributed understandings of what he was trying to say

...then he comes back with more knowledge than they assumed...they get hackles up, having made an assumption about his ignorance that was a little bit excessive..and it is on.

Forgetting the specifics, but before any alleged hate speech (I missed it), GWG spoke about some stuff, regulars did not initially see that he had some bona-fides (agreeing or not) and the rest was a retrenchment from that initial imbroglio.


This pretty much sums it up. As for GWG, IMHO he was underestimated with regard to potential contribution. His attacking style no doubt turned many off, but he too was attacked regularly and often unwarranted because "he deserved it". I also missed any overt "hate speech", but on BBI, falling into the category of hate speech is all too easy. I disagreed with much of his philosophy and stated views, but coupling those views with an aggressive attacking style doomed him as a poster.

The attacks are an unfortunate aspect of BBI, but valuable discourse still manages to come through.
well, I 'hate it ' when the soda says 'clementine'  
idiotsavant : 5/28/2015 11:03 am : link
but its mostly plain old orange, you know?

but...what about all these charts showing more jobs during the whole inflationary high rates periods? somebody please splain me all those charts, and how we get good old fashioned wages back into the business cycle.
so, if you go to the last chart I put up.  
idiotsavant : 5/28/2015 11:28 am : link
I am uninformed, to be sure:


But it looks like what you have is prices going up, without being pushed by wages going up (or is it wages staying very low despite prices going up that might pull them?).

I guess, what you have is that wages in -China- went up around that time,

so WE, big consumers of Chinese products, felt a form of 'inflation' (another wacked out unit of measure that probably needs to be looked over and broken down) while not enjoying the benefits of increased participation rates and weekly wage raises that used to correlate with it in days of yore, when we had a more singular economy.

But, then, people are probably arguing right now about what 'inflation' even -means- anymore when things and this discombobulated.

In addition, prices still increase on speculation, if you look at the increased inclusion of / access to commodities into markets (or, more accurately, to more financial groups and institutions due to IT and trading technology, as well as the continued huge un-deployed pool of cash which drove the distortions in the first place) as well as speculation on housing a such.

so, local demand was not pushing that, thus the gap at he end of the chart, which looks new.

shot in the dark that its discussed here  
idiotsavant : 5/28/2015 11:31 am : link
http://www.safehaven.com/article/21553/you-cant-have-one-without-the-other
River  
Bill2 : 5/28/2015 11:55 am : link
Trust me on this. I have no Axe to grind but the assumption he was chased off for one bad text and had things to contribute.

He did have factoids he wove into opinions ( at this point he is no different than any poster at all) on matters which demand give and take for all opinions on the matters discussed have a 35% or greater " uncertainty" factor.

Therefore certainty and dismissiveness are unwarranted. Nor did he discuss topics he often went after poster after poster on threads which later then drew the ire of many. Intolerance plus posting in rage plus drunk posting ( his words) plus contempt made it very hard to overcome the later one or two threads which showed potential. I submit that not all the interactions were seen before erased by the original OP.
I don't have difficulty with difficult posters. I do with those who fault or not presence wrecks most threads they are on. We had more than one very well informed but unable to discuss kinds of posters over the years. It's a shame. Best hope is a time and self awareness next time around. All imho. After all this is unfair for he is not here. Obviously we all wish to take back things we posted and times we were not at our best. Fair is fair. But for this season as a group we did not lose on the exchange. Again. Most of the time there is a next time.

Hope you are well my friend

Plenty of people lose their cool from time to time...  
Dunedin81 : 5/28/2015 12:01 pm : link
but some people are just perpetually unpleasant.
1976 is a year you guys could explain as well  
idiotsavant : 5/28/2015 12:02 pm : link
it looks like the seeds of the 1980s boom may have been laid in 1976, as some of [what is known as formerly the middle class] saw what was happening and decided to get off the train of wage and salary and start investing. (I am totally ignorant and probably reading the last chart totally wrong)

but...not without context that there was a base of economic activity very different from the one that exists today, (participation and % of wages in GDP)

so, what is the view on that, do we have to go back to 1970, to get to '76 to get to '80?
As far as your shot in the dark is concerned, idiotsavant...  
manh george : 5/28/2015 12:08 pm : link
My straightforward view is that anyone who doesn't consider the impact of accelerating technological change (ATC--my phrase) on the way the numbers work is simply behind the times. Don't think it matters much? Fine, make your case. But ignoring the topic suggests that you just haven;t studied it at all.

Btw, I had recommended "the Second Machine Age" as a book early last year that linked ATC to some of the economic issues. My new favorite is "The Rise of the Robots: Technology and the Threat of a Jobless Future." Lousy title, excellent book. It really isn't mostly about robots. It's about all of the main ways that ATC is likely to affect jobs and the economy. It was on the cover of the Times Book Review a couple of weeks ago.

Linked.


Link - ( New Window )
Link to the Book Review.  
manh george : 5/28/2015 12:08 pm : link
Here.
Link - ( New Window )
1970 to 1976  
idiotsavant : 5/28/2015 12:09 pm : link
may also explain why baby boomers are so insufferable,

since they 'thought their shit did not smell' during that long expansion in weekly earnings growth, a long party basically, in some sectors, which has been nudged into forgotten history by simplistic views on econ and society then.

(the death of the inner city and all that, true, truem but...the 1970's were much much more complex than that, it was gravy for many, again, the chart and anecdotally from family, and no, we were not rich)
MG, not at all, I get it and totally believe it, the tech effect  
idiotsavant : 5/28/2015 12:15 pm : link
I just think that there are factors AS WELL that people are not looking at.


The tech thing...it has made the investors themselves much, much much, more efficient and far reaching as well (in the lnk) ...so that is a multiplier effect on commodity prices...

as well as a cause of the decoupling of wage from GDP that I think you mentioned.

I am trying to learn about the upside of the rate increases in a broad, no values attached kind of way.

Or at least look at historic correlations, and have you guys parse the correlations from causes.
RE: River  
River Mike : 5/28/2015 12:18 pm : link
In comment 12304162 Bill2 said:
Quote:
Trust me on this. I have no Axe to grind but the assumption he was chased off for one bad text and had things to contribute.

He did have factoids he wove into opinions ( at this point he is no different than any poster at all) on matters which demand give and take for all opinions on the matters discussed have a 35% or greater " uncertainty" factor.

Therefore certainty and dismissiveness are unwarranted. Nor did he discuss topics he often went after poster after poster on threads which later then drew the ire of many. Intolerance plus posting in rage plus drunk posting ( his words) plus contempt made it very hard to overcome the later one or two threads which showed potential. I submit that not all the interactions were seen before erased by the original OP.
I don't have difficulty with difficult posters. I do with those who fault or not presence wrecks most threads they are on. We had more than one very well informed but unable to discuss kinds of posters over the years. It's a shame. Best hope is a time and self awareness next time around. All imho. After all this is unfair for he is not here. Obviously we all wish to take back things we posted and times we were not at our best. Fair is fair. But for this season as a group we did not lose on the exchange. Again. Most of the time there is a next time.

Hope you are well my friend


Thanks Bill. I agree that threads he was on usually went downhill rapidly, and perhaps BBI is better off without him. And also perhaps he was the main reason the threads went downhill, but he wasn't the only reason. I hate to see anyone banned, especially if they've shown some value potential. But I concede that his approach made banning inevitable. "certainty and dismissiveness are unwarranted" ... should be in the code of conduct :)

And I am doing very well thank you. Had my best day ever flyfishing the other day. My 2 sons and I got about 80 trout on flies (obviously released), and I spent most of the time tending to my grandson while my sons fished :). I hope you are doing just as well or better!
RE: 1970 to 1976  
ctc in ftmyers : 5/28/2015 12:21 pm : link
In comment 12304193 idiotsavant said:
Quote:
may also explain why baby boomers are so insufferable,

since they 'thought their shit did not smell' during that long expansion in weekly earnings growth, a long party basically, in some sectors, which has been nudged into forgotten history by simplistic views on econ and society then.

(the death of the inner city and all that, true, truem but...the 1970's were much much more complex than that, it was gravy for many, again, the chart and anecdotally from family, and no, we were not rich)


Well if you want to go back to gas rationing and 16% mortgage rates, go for it.
hahaha, that part we all know about (downside of the '70s)  
idiotsavant : 5/28/2015 12:30 pm : link
nobody wants to explain the high labor participation rates, high component of wages in GDP and the chart I pasted in where the weekly price earnings growth is now (2012 anyway) staying steadily below the retail price index for so long, which looks unique on the chart!

I am not stating a position...looking to learn
well, OK to ba fair MG showed me the tech effect  
idiotsavant : 5/28/2015 12:33 pm : link
on the % of wages in GDP...I get that.

Obviously, the 1970s is complex and yet, again, internet search, it is just so typical to write it off as a sucky time and not give the complexity of the high rates and job growth.

''I just want a lesson ;-{ ugh''

hahaha (crying / whining sounds, hehe)
idiotsavant, I wasn't referring to you.  
manh george : 5/28/2015 12:34 pm : link
I was referring to the author of the article you linked.

The response of an awful lot of economists to this topic is "la la la la la, I can't hear you."



there must be a psychological component  
idiotsavant : 5/28/2015 12:42 pm : link
to how people invest their time and adjust lifestyles and worldview that we are overlooking, that is correlated with savings return rates, job growth rates, national debt...all that
MG, thanks  
idiotsavant : 5/28/2015 12:44 pm : link
now we are getting somewhere. expound on that before I start whining again.

Lets go full contrarian here, yo.
RE: RE: 1970 to 1976  
njm : 5/28/2015 12:50 pm : link
In comment 12304225 ctc in ftmyers said:
Quote:
In comment 12304193 idiotsavant said:


Quote:


may also explain why baby boomers are so insufferable,

since they 'thought their shit did not smell' during that long expansion in weekly earnings growth, a long party basically, in some sectors, which has been nudged into forgotten history by simplistic views on econ and society then.

(the death of the inner city and all that, true, truem but...the 1970's were much much more complex than that, it was gravy for many, again, the chart and anecdotally from family, and no, we were not rich)



Well if you want to go back to gas rationing and 16% mortgage rates, go for it.


Along with a recession that many of today's recent college grads would find (except for the level of student loans) very familiar. Boomers who got out of school from '74-'76 (and in '79-'83 outside of the 70-76 time frame)learned in no uncertain terms that their shit did indeed stink.

And as far as labor participation rates go, I'm sure it looked better than the previous 20 years, but that's mostly due to the increasing participation of women.
I feel like that last chart shows that we are doomed  
idiotsavant : 5/28/2015 12:51 pm : link
when the retail price index no longer relates to the average weekly wage growth, we may have become the punks of the universe

- if a guy in China gets a pay increase, and we pay increased retail prices, and get zero benefit to OUR wages.

its been talked bout, but now the chart is starting to look scary. that seems new.
the 9:17  
idiotsavant : 5/28/2015 1:02 pm : link
.
idiotsavant  
ctc in ftmyers : 5/28/2015 1:06 pm : link
You asking good questions. I'm learning too. That's why I enjoy these discussions.

If the economy was simple, there never would be these discussions.
flying blind  
idiotsavant : 5/28/2015 1:11 pm : link
but the charts don't always reflect the narrative.

I am NOT trying to be a Carter apologist, haha, its just looking at charts and trying to get people to 'splain 'em.
.  
Bill2 : 5/28/2015 1:57 pm : link
River. Your comments about fly fishing hurt and make me wonder what I am doing instead of the great things in life.

And agree that a thread takes a village.

Savant: to understand what you are looking for you have to go back to the unholy hell of debt and transfer payments set up in the sixties, the peace effect post Vietnam which set up the inadequate ability to respond even if we wanted to response to the early round of Me nonsense of the late 70s. But most of all you would have to look up the impact on certain industries by the oil shocks of the seventies. The impact on certain but not all industries was significant. However the timing of the impact is delayed om the industrial side for the operational logistics of distribution grind down over 18 to 36 months. And once lost logistical scale needed for profits takes a long time to reach. For example, in the 1970s food, textiles, Hardware shoes and health and beauty aids were widely distributed from many small retail outlets. The top 25 companies dedicated to food outlets distributed 26% of the food. New costs of fuel took out whole middle layers until now the top 25 companies distribute almost 75% of the food. Wal mart was a major master of cross docking, or technology, JIT instead of inventory backwards integration, Ppf pricing power calculations, location operation research and all of the other innovations possible before 1973 but not yet necessary. Ditto fuel eating industries like steel who absorb enormous amount of electricity and then cost a lot to transport. The successive and lesser shocks to the operational profit economics of many an unconsolidated supply chain caused by shocks of the 1979, 1981, 1986 and 1990 me production behavior made technologies long ago developed by the Navy in ww2 useful for other reasons in the 1970 and 80s. I think you will find the factor analysis and causation vs correlations very tricky. A veritable gold field for those selling magic bullets and conclusions. ..like politicians and anyone who might extract rent from the argument. There is even rent extraction in composing arguments others use. Lots of people with graduate degrees are employed for this very essential value creating purpose.
RE: idiotsavant, I wasn't referring to you.  
Dunedin81 : 5/28/2015 2:38 pm : link
In comment 12304249 manh george said:
Quote:
I was referring to the author of the article you linked.

The response of an awful lot of economists to this topic is "la la la la la, I can't hear you."




I am not as well-read on this as you, by any stretch, but I think there's a difference between those who are truly unwilling to acknowledge something different might be happening and those who are apt to contextualize it amidst other, more traditional explanations for particular phenomena. And frankly, it's a bit fatalist to suppose that things like increased inequality are essentially inevitable. I think, perhaps I prefer to think, that inequality is at least partly attributable to various policy decisions that we've made, and thus that can be changed or reversed.
Inequality is cultural  
WideRight : 5/28/2015 3:59 pm : link
And cultures can change thier values, except in assymetric democracies where laws like Citizens United permit assembly of fortified voting blocs.
Inequality - ( New Window )
bill, was out for a bit, ugh if only what i was doing was this fun  
idiotsavant : 5/28/2015 5:01 pm : link
so...of course it would take me a while to really digest all that, your discourse on the 1970's.

I guess the, getting back to this, what is the language, the economy, the flow of capital, disposable income spending, was less clumpy back then,

In one sense, anyway,

with so many individuals (wage earners) able to spend with discretion, and such a more diverse retail environment in terms of food and stuff, (that you mentioned),

but then, on the other hand, there was probably much less capital, (a much, much less vibrant and complex and able investment world), so there is that, of course, nobody wants to loose the good part; the incredible fluidity and awesome ability of the modern capital markets to fund growth

...IF and WHEN there is any real demand uptick.

Of course, if most people cannot figure out this stuff, how can I?

...so I am not going to try to dump on the whole revolution in investable money, on the contrary.

BUT...what Dune may have been getting at, we cannot just be fatalistic about the demise of the wage earner as part of the business cycle.

I guess the conversation starter would be:

"IS it a worthwhile goal to increase the % of GDP that comes from simple wage earnings and IF so HOW to do it"

(and it does not appear that such is even a goal right now)

Enter the populists (getting back to Santorum)...or pretenders at populism, in both parties, getting back to Bills point about people who make rent on data, if I got that correctly.

and

"How to make it more likely that people who rely only on wages will have disposable income"

"...IS that a worthy goal, and IF so how to make steps in that direction?"

Perhaps this is what Dune meant as well, it is a bit cynical to just assume that people proposing policy changes are just in it for short term gain or political hay

...there are plenty of people out there who would love to make a living by dint of labors alone once again, and it would do wonders for the capital markets, since they would have a less abstract place to put money...regular old growth lending to start...then...more of all the above.
savant  
Bill2 : 5/28/2015 5:30 pm : link
I don't know what to grab onto from that post. But I will say these two items:

Capital markets made much more sound and long term investments of the accumulated national wealth so future generations were more likely to be stable and generational wealth transfer at a national level was enormously safer with checks and balances and hugely different rewards and time lines and perspectives.

All you need as factoid evidence is the day the partners at Goldman went from personal wealth based on the sound investments of the partners money at the time they retired to annual bonuses based on returns from transactions completed that year using other people's money. 1986. If you seriously think that politicians and the FIRE sector makes better decisions now than 1810 to 1985 I don't know where to start. "Sophisticated sounding" and "sound investment" begin with s but that's about it. Imho.
I never understood Goldman going public  
WideRight : 5/29/2015 8:10 am : link
Some of the smartest guys in the world going public, selling "high" when the Dow was somewhere around 2000!

I recall each partner got about 50M, which is chump change compared to the opportunites lost. 50M shuts alot of people up, but I still bet, in a moment of honesty, they would admit the fallacy of their short-sightedness.
Wide Right  
Bill2 : 5/29/2015 8:20 am : link
There is a cadre of senior partners ex and current that see it that way. And see the phenomenon as the national interest tragedy it is. In every profession the very very top fraction are truly remarkable and more like others at the very top of their field than the rung below.
Actually, the Dow was around 10,000 ....  
njm : 5/29/2015 8:25 am : link
when Goldman went public.
And an apology  
Bill2 : 5/29/2015 8:32 am : link
Wide Rights point about the market trading point reminded me that I got Solomon and Goldman mixed up. First the Philbro merger and then going public in as pure memory hopes is right in 1986; showed a path the old line IB'S could not resist, claiming Goldman in 1999. What I do remember is being overseas and seeing the announcement in the WSJ and knowing that day was going to come back and haunt us all. Misalignment and twists on John Nash work and computing power all formed a witches brew of "innovation" and inability to manage risk inside and outside institutions that the nation relies on to be the adults in the room.

Bill2, thanks for showing me some stuff  
idiotsavant : 5/29/2015 8:38 am : link
your last post prior had me understanding you much better! :-)
Bill  
njm : 5/29/2015 8:42 am : link
Given Goldman held out as long as they did, I wonder if the repeal of Glass Steagall was the game changer.
that must have seemed a crazy moment, but what about this:  
idiotsavant : 5/29/2015 8:46 am : link
trend:

(dailyfinance '1.2 quadrillion derivatives market dwarfs world GDP)



''One of the biggest risks to the world's financial health is the $1.2 quadrillion derivatives market. It's complex, it's unregulated, and it ought to be of concern to world leaders that its notional value is 20 times the size of the world economy. But traders rule the roost -- and as much as risk managers and regulators might want to limit that risk, they lack the power or knowledge to do so.

A quadrillion is a big number: 1,000 times a trillion. Yet according to one of the world's leading derivatives experts, Paul Wilmott, who holds a doctorate in applied mathematics from Oxford University (and whose speaking voice sounds eerily like John Lennon's), $1.2 quadrillion is the so-called notional value of the worldwide derivatives market. To put that in perspective, the world's annual gross domestic product is between $50 trillion and $60 trillion.''
so, putting that into USA GDP perspective  
idiotsavant : 5/29/2015 8:52 am : link
you have wages as a smaller and smaller ...and smaller part of GDP,

and GDP in turn is dwarfed by the derivatives market

(which ....who the fuck knows if and how that is counted back into the above or if it should be, or where, or why or anything)

holy crap this is a very weird world we live in, at what point are we just in a virtual reality built by and for investors?

maybe I should wear a clown hat today ...all day...in public, as an expression of how absurd this seems
or maybe it is just a symptom that there is still a huge  
idiotsavant : 5/29/2015 8:56 am : link
and I mean huge pool of cash, with no where to go (new factories, not needed right now, thank you) which puts the whole cheap money thing in perspective, as if we need any more of that medicine.

but...are we in position for the other medicine, or caught in a trap?
.  
idiotsavant : 5/29/2015 8:58 am : link
njm  
Bill2 : 5/29/2015 9:00 am : link
I think so. I was not close to privy to the internal dialog but I imagine it set up a run on talent for them if they did not change with the writing on the wall. In the end the glib but second tier of their potential ( Rubin, Corzine) won outside in. Imho.
Smarter people can explain this better...  
Dunedin81 : 5/29/2015 9:05 am : link
but the idea that derivatives have "quadrillions" in value is to misapprehend what derivatives are. They are options, often mutually conflicting (an option to buy a stock at a set price and an option to sell it at a lower price are not going to be exercised in concert because the incentive to do one is necessarily the rationale for not doing another - the movement of the stock price up or down).
maybe what we really need to do is build a colony on the moon  
idiotsavant : 5/29/2015 9:06 am : link
it would be like a shared growth economy, all sorts of manufacturing needed and expanding every which way. 100% employment.
ok, Dune, but, nevertheless  
idiotsavant : 5/29/2015 9:12 am : link
even if it is 80% smoke (or, each dollar having 4x possibilities attached to it) , there is real money attached to the market in some way, shape or form,

and, to me it would not have been invented if there was a better place for that cash. so it shows that the pool of cash is still plenty big enough. what we lack as a planet is demand for real goods growth in line with the pool of cash.

so a % that cash can be deployed to people who build robots

(I am loosing it)
RE: ok, Dune, but, nevertheless  
Dunedin81 : 5/29/2015 9:14 am : link
In comment 12305528 idiotsavant said:
Quote:
even if it is 80% smoke (or, each dollar having 4x possibilities attached to it) , there is real money attached to the market in some way, shape or form,

and, to me it would not have been invented if there was a better place for that cash. so it shows that the pool of cash is still plenty big enough. what we lack as a planet is demand for real goods growth in line with the pool of cash.

so a % that cash can be deployed to people who build robots

(I am loosing it)


Oh it is serious, I'm not suggesting that they present no risk, I'm just saying that looking at the notional amount of all the derivatives outstanding in the world at a given time is something people do when they want to sensationalize, and I don't think that's what you're trying to do.
to me, complexity exists for a few reasons  
idiotsavant : 5/29/2015 9:17 am : link
1. to hide something

2. because simplicity was not enough (the real econ shrank relative to the un-deployed pool of cash, loans and straight equity being relatively simple ways to deploy in growth)

3. some people love complexity (but would that be enough to make it happen without the top 2 reasons?)
Dune, I am ignorant so I just took the article at its face value  
idiotsavant : 5/29/2015 9:23 am : link
but, the trend I am pushing is to look at

cheap money vs expensive money

and the effects of that on the wage earners sector.

and the complexity of the derivatives (and size, in some degree) seems like a distortion in and of itself, that may be rhetorical,

at least, a symptom of that the real econ never caught up even closely to the pool of cash (which the cheap money was supposed to increase, that pool of cash)

so, for people in the expensive lending and direct investment world  
idiotsavant : 5/29/2015 9:31 am : link
the problem has not been raising capital into your fund, the problem has been, for every $10m deal, there are 8x that $10 looking to get in.

so, people keep inventing new things to do with money (the 'distortions') and waiting for someone to come knocking who wants to build a factory....and waiting...and waiting.

(why everyone loves Elon Musk, and why not love him?, we need 10,000 more like that)
RE: Actually, the Dow was around 10,000 ....  
WideRight : 5/29/2015 9:34 am : link
In comment 12305459 njm said:
Quote:
when Goldman went public.


Thank you for the correction. I forgot the year as well.
Goldmans is up 180% since going public 5/14/15  
WideRight : 5/29/2015 9:47 am : link
About twice that of the Dow.

So that ~50M capital each partner took would be 100M. Thats an awful lot of $$ for smart people to give up. Along with future earnings and appreciation.

And its likely if they had remained private during the financial crisis they could have profitted considerably more. A major opportunity lost.
5/14/99  
WideRight : 5/29/2015 10:27 am : link
.
RE: Goldmans is up 180% since going public 5/14/15  
njm : 5/29/2015 10:33 am : link
In comment 12305607 WideRight said:
Quote:
About twice that of the Dow.

So that ~50M capital each partner took would be 100M. Thats an awful lot of $$ for smart people to give up. Along with future earnings and appreciation.

And its likely if they had remained private during the financial crisis they could have profitted considerably more. A major opportunity lost.


I wouldn't be so certain. The unanswered question is whether the management team would have been the same one that would have run the show if they had stayed private.
Goldman got incredibly lucky during the Financial Crisis...  
manh george : 5/29/2015 11:07 am : link
as well as probably crossing the line ethically and legally, and to this day, people seem to forget that.

Goldman was the biggest player in AIG CDS, that Geithner decided had to be paid off 100 cents on the dollar. The case is also made in the linked article that Goldman bought protection on AIG while holding information that the sellers didn't have.

If they didn't come out of that mess smelling like a rose, the decision to go public and let a lot of people cash out would look less dumb.

To be sure, Goldie has been a kickass competitor since then. But they could easily have fallen into a deep, dark abyss if Geithner wasn't so chickenshit.

In addition, of course, a confluence of events since 2010 have led to an impressive rise in equity prices which goosed the value of the firms with the most leveraged exposure to that rise. Goldy would be high on that list. It's not reasonable, imo, to compare boomtime stock value on GS to what was going on when they went public.
Link - ( New Window )
My speculation was based on the fact that  
WideRight : 5/29/2015 11:53 am : link
as a private concern, their liquidity issues would not be subject to the same regulations and reporting. They could have doubled down.

The point about AIG is spot on, but would have been handled the same.
Not sure why I'm wasting my breath...  
baadbill : 5/31/2015 11:50 am : link
but happened to be reading "Winning the Loser's Game" by Charles Ellis, 5th Edition - and came across the following excerpt on page 92:

Quote:
We may be able to see that in theory our long-term interests are best served by lower stock prices, so we can all buy at bargain prices. But who among us can honestly say that he or she is delighted by falling markets? And who does not feel a warm glow of affection for stocks and markets that have gone up, even though we know it means that stocks are more expensive to buy and future rates of return on additional investments at these price levels will surely be lower?

By contrast, who among us would close our pocketbook and turn away from the store that puts its most attractive wares on sale at 10, 20 or even 30 percent off its recent prices? None of use would say, "I don't want to buy these things when they're on sale; I'll wait until the price goes back up and buy then." But that's exactly how most of us behave toward investments.

When the market drops - putting stocks "on sale" - we stop buying. In fact, the records show that we even join in the selling. And when the market rises, we buy more and more enthusiastically. As Jason Zweig of Money magazine puts it, "If we shopped for stocks the way we shop for socks, we'd be better off." We are wrong when we feel good about stocks having gone up, and we are wrong when we feel bad about stocks having gone down. A falling stock market is the necessary first step to buying low.
That you think  
Rob in CT/NYC : 5/31/2015 12:03 pm : link
That supports your initial post shows you have no idea what the difference of opinion even was.

Dollar cost averaging is a perfectly acceptable strategy, which of course your last post suggests (correctly) should not be abandoned based on sentiment. It in no way endorses your assertion that a declining market every year during your accumulation phase is a good thing. In fact, you suggested it was the preferred outcome.
RE: That you think  
baadbill : 5/31/2015 12:12 pm : link
In comment 12308508 Rob in CT/NYC said:
Quote:
That supports your initial post shows you have no idea what the difference of opinion even was.

Dollar cost averaging is a perfectly acceptable strategy, which of course your last post suggests (correctly) should not be abandoned based on sentiment. It in no way endorses your assertion that a declining market every year during your accumulation phase is a good thing. In fact, you suggested it was the preferred outcome.


The quoted section isn't discussing dollar cost averaging. It is talking about the accumulation phase of investing (of course, dollar cost averaging is undoubtedly a part of regular accumulations - but it is most certainly NOT the focus of the quoted section.

Go ahead and ignore the following quote:
"... in theory our long-term interests are best served by lower stock prices..."

"We are wrong when we feel good about stocks having gone up, and we are wrong when we feel bad about stocks having gone down. A falling stock market is the necessary first step to buying low."

My first post was that long term investors should cheer when their portfolios get devastated. I stand by that. The quoted section stands by that. And it isn't surprising to me that you don't understand that. But I'm not posting to you. I'm posting to all those who read your posts to warn them to ignore you and do their homework [to read the ACADEMIC studies on their own] before listening to you or me.
Again.  
kicker : 5/31/2015 12:33 pm : link
Holy fuck...
By the way, I think most people should listen  
kicker : 5/31/2015 12:36 pm : link
to a person with a proven track record in finance, rather than a guy who read a book.
RE: By the way, I think most people should listen  
baadbill : 5/31/2015 12:43 pm : link
In comment 12308532 kicker said:
Quote:
to a person with a proven track record in finance, rather than a guy who read a book.


That shows how much you know. EVERY academic study has proven that the one place you should NEVER trust your money is with anyone who works for a living on wall street. Active management of stock mutual funds - no less buying and selling of individual securities - is a losers game. After transaction costs, it has been proven over and over and over again that wall street "experts" can't beat the market and do worse than someone selecting stocks blindfolded throwing darts at the financial section.

So for anyone reading this thread, it is important for their own financial future to read the one group of people who have no axe to grind - the academic community (and the first thing the academic community will tell you is TO PUT YOUR HANDS ON YOUR WALLET AND TOTALLY IGNORE ANYONE WORKING FOR A STOCK BROKERAGE OUTFIT OR OTHERWISE WHO PRETENDS TO HAVE YOU BELIEVE YOU CAN BEAT THE MARKET BY PAYING HIM)
Since I'm part of the academic community, they  
kicker : 5/31/2015 12:50 pm : link
shoud listen to me?

Good to know.

By the way, you realize that finance is much larger than active money management? Or, is that not in the books?
RE: Since I'm part of the academic community, they  
baadbill : 5/31/2015 1:01 pm : link
In comment 12308543 kicker said:
Quote:
shoud listen to me?

Good to know.

By the way, you realize that finance is much larger than active money management? Or, is that not in the books?


No. They should not listen to you. They should listen to Burton Malkiel. David Swensen. Eugene Fama and certified financial planners approved by Dimensional Fund Advisors.

And Charles Ellis. And the quotes from Ellis I just made. This is pretty simple stuff. Markets to up and future returns are lower. Markets go down and future returns are higher. Buy an entire stock market index and add to it regularly for your 60 years of investing (assuming you live until you are 80) and you will beat every single wall street "expert" in existence. And, of course, you will do better if you happen to CHEER market drops and buy more on the way down - and DECRY new market highs which mean every penny invested at that time will end up being your worst performing investment over your lifetime.

But they most certainly should NOT listen to you. That's clear from your posts on this thread. You make fun of the 100 or so books I've read on investing - but it's pretty clear that you have read none of them.
Haha. Sure  
kicker : 5/31/2015 1:07 pm : link
I'll take my track record. But really don't care if a dolt believes me. Your posts talk for themselves.
Baadbill - There is nothing wrong with being ignorant  
Rob in CT/NYC : 5/31/2015 1:11 pm : link
There is great deal wrong with aggressive stupidity, and it is very unrewarding to try and discuss anything with an individual such as yourself - life is too short.

Here is wishing you success and happiness.
And to clarify - I am not suggesting that anyone should be  
baadbill : 5/31/2015 1:20 pm : link
100% invested in stocks ... my reference to a total stock market fund was to make the simplistic point - irrefutable point - that index funds beat actively managed funds hands down. And that there is nobody in history who has been able to successfully time the market.

So index funds are the only way to invest. And appropriate asset allocation is the single most important determinant of return over a long period of time.

And that the second most important determinant of return are fees. The more you pay the lower your returns. Index funds win for that basic reason. Vanguard's total stock market fund can be held while paying 5 basis points per year. No stock picker is going to beat that over time.

The last thing is luck. When you are born. What the returns are like during your 40-60 years of investing. And are you investing during market highs or market lows.

Today - for example - one of the worst times to be investing in the stock market. SCHILLER PE is over 27. It's only been there three other times. I'll let you guess when those three times have been. This is a great market for those who are already retired and living off of their portfolio - because the growth has been great for them.

For everyone else (i.e. those of us still accumulating assets), these prices suck. Future returns for money invested now are miserable - by definition. It is only when the market falls (and regression to the mean requires that it will fall), that investors will be able to once again have hopes for high future returns from the stock market.

Worse, right now the bond market is at an all time low. So there aren't really a lot of good alternatives. But, I digress. The real point is that the OP talked about HIGH PRICES being good and LOW PRICES being bad. The truth is that is a misunderstanding of basic investing principles. It is a true statement for those who have already retired and are SELLING to live off their assets. But it is a FALSE statement for the rest of us who are still BUYING.

But - again - my time here isn't being spent for you. For you, I leave you to go ahead and put your money into stocks now. Perfect time for YOU to invest. But for the rest of BBI, read the academic literature. Understand that high markets should NOT make you happy. LOW MARKETS should make you happy and are the time to invest.
Page 122 from  
baadbill : 5/31/2015 8:37 pm : link
Quote:
Question: If you had your choice, which would you prefer?

CHOICE A: Stocks go UP - by quite a lot - and stay UP for several years.

CHOICE B: Stocks go DOWN - by quite a lot - and stay DOWN for several years.

Make your choice before you look at the next page.


Without looking ahead, which did you choose? If you selected choice A, you would be joining 90 percent of the investors - individual and professional - who've taken this test. Comforted to know that most pros are with you? You shouldn't be. Unless you are a long-term seller of stocks, you would have chosen against your own interests if you chose A.

Here's why. First, remember that when you buy a common stock, what you really buy is the right to receive the dividends paid on that share of stock. Just as we buy cows for their milk and hens for their eggs, we buy stocks for their current and future earnings and dividends. If you ran a diary, wouldn't you prefer to have cow prices low when you were buying so that you could get more gallons of milk for your investment in cows?

The lower the price of the shares when you buy, the more shares you will get for every $1,000 you invest and the greater the amount of dollars you will receive in future dividends on your investment. Therefore, if you are a saver and a buyer of shares - as most investors are and will continue to be for many years - your real long-term interest is, curiously, to have stock prices go DOWN quite a lot and stay there so you can accumulate more shares at lower prices and therefore receive more dividends with the savings you invest.

Thus, the right long-term choice is the counterintuitive choice B. This can be the key insight that may enable you to enjoy greater success as an investor and greater peace of mind during your investing career. You may even learn to see a benefit in bear markets. If you're really rational, you will.
oops ... from Winning the Loser's Game by Charles Ellis  
baadbill : 5/31/2015 8:38 pm : link
.
What the above doesn't discuss...  
manh george : 5/31/2015 9:15 pm : link
is WHY the market is up a lot or down a lot for several years. And, of course, it is discussed in the context of a market at an all-time high.

If the stock market goes down and stays there because the economy goes into the shitter and stays there, your outcome won't be so hot. Promise.
RE: What the above doesn't discuss...  
baadbill : 5/31/2015 9:22 pm : link
In comment 12308871 manh george said:
Quote:
is WHY the market is up a lot or down a lot for several years. And, of course, it is discussed in the context of a market at an all-time high.

If the stock market goes down and stays there because the economy goes into the shitter and stays there, your outcome won't be so hot. Promise.


The economy being in the shitter is usually the cause of the market being down for a sustained period of time. While that may not be a great thing for your job - or for your income - from the standpoint of investing - as a BUYER of stocks, you are better off when the market is DOWN.

You still don't get it. That's ok. As the above quote says, you can make yourself feel better by the fact that 90% of "professionals" (i.e. wall street salesmen) don't get it either. But I'm not going to argue with you. You can argue with Charles Ellis - only one of the most widely recognized and preeminent investment advisors around.
Nope, wrong again.  
manh george : 5/31/2015 9:40 pm : link
You are assuming a cyclical decline in the economy. Technically based analysts/strategists, including the writer of you favorite book, ALWAYS assume that the economic condition which led to the market drop is cyclical, not structural.

As has been pointed out by many economists/strategists who go beyond a technically based view, in the current environment, I can think of at least three different environments in which the economy doesn't bounce back for a long time, keeping the stock market down with it--perhaps not until after your retirement. Then how do you market time?

I can also envision a scenario whereby the market never takes a really substantial dip from here, so you never get in.

I get where you are coming from. I also know that it is inconceivable that you will ever comprehend where I am coming from, so, like Rob and others who already understand market concepts from experience rather than reading a book, I'm done.
Oh, and a-hole...  
manh george : 5/31/2015 9:42 pm : link
I'm an II ranked market strategist, not a salesman. I just cut my estimate of your IQ by 20 points because you don't understand the difference.

NOW I'm done.
RE: Nope, wrong again.  
baadbill : 5/31/2015 9:42 pm : link
In comment 12308891 manh george said:
Quote:
You are assuming a cyclical decline in the economy. Technically based analysts/strategists, including the writer of you favorite book, ALWAYS assume that the economic condition which led to the market drop is cyclical, not structural.

As has been pointed out by many economists/strategists who go beyond a technically based view, in the current environment, I can think of at least three different environments in which the economy doesn't bounce back for a long time, keeping the stock market down with it--perhaps not until after your retirement. Then how do you market time?

I can also envision a scenario whereby the market never takes a really substantial dip from here, so you never get in.

I get where you are coming from. I also know that it is inconceivable that you will ever comprehend where I am coming from, so, like Rob and others who already understand market concepts from experience rather than reading a book, I'm done.


Like I said, make your arguments with Mr. Ellis, but I suspect he has just a tad more experience than you.
RE: Oh, and a-hole...  
baadbill : 5/31/2015 9:56 pm : link
In comment 12308893 manh george said:
Quote:
I'm an II ranked market strategist, not a salesman. I just cut my estimate of your IQ by 20 points because you don't understand the difference.

NOW I'm done.


Glad you're done. Everyone who works on wall street is a salesman. If you don't understand that, I don't know how you do your job. Then again "market strategist" says it all. Every academic study ever done - 100% of them - have demonstrated that it isn't possible to beat the markets. So I'm glad you have a job that is the equivalent of a newspaper horoscope. And I'm glad your so proud of being a level II horoscope creator.

I'd love to hear your explanation of what you do some day. Do you time the market? Are you a "technician" who reads charts? Are you a fundamentalist who believes he can follow a particular company or industry and "predict" it's quarterly earnings? If you are truly in the business, then fess up and admit to fellow BBI members what hocus pocus it all is. I trust you've read "Where Are All The Customer's Yachts"? Written almost 70 years ago by a Wall Street Analyst, it's still a best selling book today for good reason - he gives us the inside scoop on Wall Street.

Anyone who has any common sense would know that if you (or anyone else on Wall Street) really were able to beat the market, you wouldn't work for a living - you'd already be independently wealthy.
I co-run an academic conference in my sector--which isn't stocks, btw,  
manh george : 5/31/2015 10:22 pm : link
and there hasn't been an academic paper in my sector in the 35+ years I have been following the literature that I can't rip into shreds. Well, maybe one.

Carry on.
I don't claim to beat the market.  
manh george : 5/31/2015 10:27 pm : link
I do ok in my own portfolio, no better. I actually spend my time on my job, not on picking stocks.

I do know a rube when I see one, though.
Link - ( New Window )
baadbill  
Bill2 : 5/31/2015 10:37 pm : link
I don't invest a dime in the stock market.

Don't work on Wall Street.

Don't work in the FIRE sector at all.

Was never sold anything by anyone on WS that led me to invest in the Stock market.

Simple reason. I believe execution, not strategy, matters more in most, but not all, Companies and sectors. Therefore if I don't closely watch the execution of a direction I heavily influence...I don't invest that someone else will.

Sold my stocks given by my grandparents to buy books in college and that was the last time I made a stock transaction.

and I have more than my share of disagreements with the folks you are arguing with. and respect and consider many of them actual friends.

While Manh resorted to citing what he self identifies as authoritative....he could have dismantled your misunderstanding of "structural" by the authority of logic.

Bill, you can believe anything you wish. I don't understand why it started...sound to me like semantics and poorly written but obvious what you were saying at the core. However, you cannot rip posters for being part of the investment world and champion books by someone in the investment world. Which is it? And why rip other posters to begin with when you were the one who cited an "authority"?

You cant beat the market...unless you do it this magic way by this investment advice....but then rip all other forms of advice. Which is it? Advice from a guy or the cult of Ellis.

seems to me there is an awful lot of BBGOTFO.
RE: RE: Oh, and a-hole...  
BurberryManning : 5/31/2015 10:48 pm : link
In comment 12308901 baadbill said:
Quote:
In comment 12308893 manh george said:


Quote:


I'm an II ranked market strategist, not a salesman. I just cut my estimate of your IQ by 20 points because you don't understand the difference.

NOW I'm done.



Glad you're done. Everyone who works on wall street is a salesman. If you don't understand that, I don't know how you do your job. Then again "market strategist" says it all. Every academic study ever done - 100% of them - have demonstrated that it isn't possible to beat the markets.


First of all, which market or markets are you referring to? "Beat" the market over what time period? And in what way? In absolute terms? In risk-adjusted terms? Are you claiming that the majority of active managers cannot consistently outperform a benchmark or are you absolute because we can identify managers that have consistently outperformed their respective benchmarks in many ways across various asset classes.

And please understand that there is substantially more to Wall Street than asset management, and there is much more to asset management than beating an index or benchmark, and there is also more to asset management than large cap domestic equities.

It's simply become a mind-numbing discussion and the irony of the conversations in which you find yourself, citing academic figures against the posters of whom you're engaging. You're borrowing the time of folks who might share the same stage with those that you're championing.
BBGOTFO  
Bill2 : 5/31/2015 10:49 pm : link
Buying low is a better thing than buying high...if it works.

BBGCOTFO

Big Blinding Glimpse Of The Fucking Obvious.

what people are trying to tell you is that the "market" may...MAY... no longer be operating under the same rubric as it did pre 1987.

And there are lots of reasons that might be true.

Its the certainty....more valid pre 1987 than now...that people are objecting to. We all know it. Buying low is better than buying high...when it works. That's your argument. Until you understand and can accept or reject other arguments and factors to consider in this version of the "market"...you are yelling with certainty to guys who a genuinely hard working to remain open and iconoclastic within their worlds and jobs. Its not the same asset class it used to be Bill. Same rules MAY not apply. No one is putting you down personally when they say that. Take care...lets all wind down on this topic. Make sense?
No, he's a simple fuck who has advocated for a potential  
kicker : 5/31/2015 11:01 pm : link
nuclear solution for "total war".

Not only is he a dipshit, but he's a dangerously simple one.
Not to Bill2  
kicker : 5/31/2015 11:01 pm : link
...
...  
kicker : 5/31/2015 11:03 pm : link
I apologize to Bill2.

I shouldn't engage with people like that. Lost causes aren't worth my time.
RE: RE: RE: Oh, and a-hole...  
baadbill : 6/1/2015 6:26 am : link
In comment 12308957 BurberryManning said:
Quote:
In comment 12308901 baadbill said:


Quote:


In comment 12308893 manh george said:


Quote:


I'm an II ranked market strategist, not a salesman. I just cut my estimate of your IQ by 20 points because you don't understand the difference.

NOW I'm done.



Glad you're done. Everyone who works on wall street is a salesman. If you don't understand that, I don't know how you do your job. Then again "market strategist" says it all. Every academic study ever done - 100% of them - have demonstrated that it isn't possible to beat the markets.



First of all, which market or markets are you referring to? "Beat" the market over what time period? And in what way? In absolute terms? In risk-adjusted terms? Are you claiming that the majority of active managers cannot consistently outperform a benchmark or are you absolute because we can identify managers that have consistently outperformed their respective benchmarks in many ways across various asset classes.

And please understand that there is substantially more to Wall Street than asset management, and there is much more to asset management than beating an index or benchmark, and there is also more to asset management than large cap domestic equities.

It's simply become a mind-numbing discussion and the irony of the conversations in which you find yourself, citing academic figures against the posters of whom you're engaging. You're borrowing the time of folks who might share the same stage with those that you're championing.


Might share the stage with those I am championing? The academic research I'm discussing have won the Nobel Prize in Economics for their research. There are several things that are pretty clear in the world of finance:

1. Over long periods of time (30-60 years), stocks will outperform bonds which will outperform Treasuries.

2. Asset allocation is THE largest determinant of performance.

3. Efforts at stock selection (as opposed to "buying the market") will decrease performance.

4. Efforts at market timing will decrease performance.

5. Relative costs between those with identical asset allocations will make huge differences in returns over a 40-60 year period of time. A portfolio with an average 1.00% expense ratio versus one with a 0.10% expense ratio will underperform by more than 50% over a lifetime.

6. Anyone who believes that there is a capability to beat "the market" (first of all, defined by me as the total stock market - whether domestic or international - but if you want to break it down into styles such as small or large or growth vs value that's fine) - nobody beats passive index funds over time - at least not statistically better than 50,000 coin flippers in Giants stadium will produce a final 10 that can flip 50 heads in a row and are obviously "very skilled coin flippers" - the only problem being NOBODY can predict who will be the ones to flip heads 50 times in a row any better than predicting who will be lucky enough to beat the market over the next 10 years).

7. Corollary to #6 is that Wall Street has always been - still is - and always will be - snake oil salesmen. It has been demonstrated over and over and over again that they cannot add value to an index based investment strategy over long periods of time, especially when expenses and transaction costs are included (as they must be). The only one making money via Wall Street are the ones working on Wall Street. That is FACT, not supposition. But they are a great marketing engine. They have to be. They are, however, slowly losing to indexing - on a very large scale.

Now, Wall Street does produce efficient markets and for that they are very necessary. But overall their take for acting as brokers between buyer and seller is outrageous (and much much worse in bonds than in stocks). Other than providing an orderly market, they are overall a negative force for the economy and certainly extremely dangerous to the pocketbooks of individual investors, especially the uneducated who have never be trained or educated in the world of investing and simply think that a stock brokerage house is that place one goes to invest.

Funny how I start my involvement in this thread with a simple statement - trying to respond to the OP by stating that in general terms an investor in the accumulation phase should be CHEERING lower prices - and these so called "experts" on here laugh at the thought. Then I come across quotes in a book I happen to be reading (not surprising because I've read similar things in virtually ever academic book ever written on the subject), so I quote directly from a book that is considered a top 10 book on investing by most lists I've seen - and then a second quote that is even more clear - and suddenly the attack is upon the author of the book - who is simply repeating the identical themes I've read over and over again by Nobel Prize winning authors.

And - the concepts I quoted - really aren't that difficult to grasp - yet these so called "experts" can't accept the quotes - refuse to acknowledge the direct simplicity of what is being proposed. And most amazing of all - the quotes don't propose DOING anything. They simply state that when the market goes down, have a view point that it is a buying opportunity. The quotes are nothing more than recommending an attitude to control your emotions when stocks are high versus when stocks are low. Yet people here are so emotionally charged, they attack an author's recommendation for what attitude investors should have.

A pretty impressive reaction by these BBI "experts", I must say.
RE: baadbill  
baadbill : 6/1/2015 6:44 am : link
In comment 12308950 Bill2 said:
Quote:
I don't invest a dime in the stock market.

Don't work on Wall Street.

Don't work in the FIRE sector at all.

Was never sold anything by anyone on WS that led me to invest in the Stock market.

Simple reason. I believe execution, not strategy, matters more in most, but not all, Companies and sectors. Therefore if I don't closely watch the execution of a direction I heavily influence...I don't invest that someone else will.

Sold my stocks given by my grandparents to buy books in college and that was the last time I made a stock transaction.

and I have more than my share of disagreements with the folks you are arguing with. and respect and consider many of them actual friends.

While Manh resorted to citing what he self identifies as authoritative....he could have dismantled your misunderstanding of "structural" by the authority of logic.

Bill, you can believe anything you wish. I don't understand why it started...sound to me like semantics and poorly written but obvious what you were saying at the core. However, you cannot rip posters for being part of the investment world and champion books by someone in the investment world. Which is it? And why rip other posters to begin with when you were the one who cited an "authority"?

You cant beat the market...unless you do it this magic way by this investment advice....but then rip all other forms of advice. Which is it? Advice from a guy or the cult of Ellis.

seems to me there is an awful lot of BBGOTFO.


Couple of corrections:

1. I try to avoid anyone involved in the investment world if their employer sells a product. "Advice" can come from three sources: (a) Wall Street; (b) Family and fiends or other non-experts (such as myself); and (c) the academic community. Of the three, the only one that has no axe to grind - and who will be measured by his/her peers by the purity of his/her research and analysis - is the academic community. It is where I limit my reading. And I have read it extensively. I don't suggest anyone listen to me. I only suggest they educate themselves through the academic community.

2. The books I cite do NOT propose a way of beating the market. Rather they cite the studies that have demonstrated universally that nobody can beat the market over any sustained period of time, after including expenses and transactional costs. And for the 5% that have done it over a 10-15 year period, there was no predictive tool enabling an investor to pre-identify the 5% who were about to beat the market over the ensuing 10-15 years (nor is there a way to do so today). Investing in anything other than passive indexes is a loser's game.

3. There is no such thing as a "change" in the idea that buying stocks are lower prices is better than buying stocks at higher prices. Yet people (understandably) have difficulty grasping the simple rational statement that, in general, investors are better off with a falling stock market and lower stock prices over extended periods of time during their investing lifetimes - than they are with a rising stock market and higher prices over their investing lifetimes. It is counterintuitive and my comments here on that basis investing concept - investing 101 - has been met with anger and ridicule - not unexpected from the "wall street" gang.

4. The last thing is that this isn't a discussion of philosophy. It is a discussion of statistics and math. Wall Street wants to sell people on the idea that 2+2=5. The academic community will SHOW YOU their outright lies and false advertisements. To have people doing so here on BBI is fairly outrageous. My point is that this isn't opinion. It may sound that way - that is how Wall Street wants people to react - they are threatened - but in reality it is a FACT that wall street cannot, does not, and never will add value to investors over the long term. They sell a product - charge a fee - that promises underperformance. That is not an opinion. That is a FACT.
RE: BBGOTFO  
baadbill : 6/1/2015 7:01 am : link
In comment 12308958 Bill2 said:
Quote:
Buying low is a better thing than buying high...if it works....


Bill, there is no "if". It is not mathematically possible for Investor A to do worse than Investor B when:
Investor A to buy $10,000 worth of XYZ corp at $10 per share; and
Investor B to buy $10,000 worth of XYZ corp at $20 per share; and
Then they both later sell their shares at the same price.

Investing for the long term is a straight forward activity. You are collecting shares (widgets if you want). The goal is to collect as many widgets as possible because, when you retire, you will sell those widgets at whatever price the market happens to provide for at that time. The secret is to have "collected" as many widgets as possible during your accumulation stage of your lifetime. Success will be defined by how many widgets you have collected by the time you are in your "spend down" (retirement) stage of your life.

It is a simple fact that the lower the price of widgets during the time you are accumulating them, the more widgets you will own by the time you need to sell them.
badbill  
Bill2 : 6/1/2015 7:14 am : link
One...in the narrow point about buying low in s cylical market of the time before 1987 and less and less since then you and i are in violent agreement.

citing or buying what academics say because they are academics ( as opposed to a person of any kind can and do make good arguments but most are flawed no matter what background they have) is a recipe for disaster. I don't know about you but I don't buy any argument unless I have seen a variety and I believe it for myself.

Next your comments about people in the FIRE sector amount to a prejudice. I think what you say applies to the vast majority.

Academics can make as many mistakes as anyone else when they use statistics or fail to take all in context.

If you were not prejudiced you would understand that most on here are actually telling you ( most clearly Manh) that there are now emerging reasons not to listen to wall street or your academic...both. I don't know about you but when someone argues against their own alignment ....I at least listen. You are not.

Over and out. Good luck.
BurberryManning said  
baadbill : 6/1/2015 7:21 am : link
In comment 12308957 BurberryManning said:
Quote:
First of all, which market or markets are you referring to? "Beat" the market over what time period? And in what way? In absolute terms? In risk-adjusted terms? Are you claiming that the majority of active managers cannot consistently outperform a benchmark or are you absolute because we can identify managers that have consistently outperformed their respective benchmarks in many ways across various asset classes.

And please understand that there is substantially more to Wall Street than asset management, and there is much more to asset management than beating an index or benchmark, and there is also more to asset management than large cap domestic equities.

It's simply become a mind-numbing discussion and the irony of the conversations in which you find yourself, citing academic figures against the posters of whom you're engaging. You're borrowing the time of folks who might share the same stage with those that you're championing.


Of course there is more to asset management than large cap domestic equities. Asset allocation and modern portfolio theory is not what this discussion has been about. The "point" has been a relative simple one - but counter intuitive to most people. Long term investors are going to see the markets get extremely pricey and are going to see the markets get extremely depressed. The OP seemed to believe and suggest that markets going up (i.e. today's market prices) are a good thing. I was simply pointing out that most investment theory would say that is wrong (unless you are selling your assets such as retirees). But what is amazing is that the entire debate is really pretty silly. It is about attitude. Should you feel better, as an investor, when prices of stocks you are BUYING are going down or going up? The answer is straight forward.

As for managers who have "beaten the market" and definition of market, I am speaking solely about stocks (although I would say, based upon my reading, that the same would be true for bonds or commodities too). As for stocks, if you want to start sub-grouping them into style boxes (large-medium-small or growth-blend-value), then the same truth holds. The same for sectors, health, technology. Or international. Etc.

The studies show that if you put 50,000 in Giants stadium and have them all flip a coin and then everyone who flips tails leave - after flipping a fairly large number of times, the stadium will slowly empty until there are only 5 people left. By definition those 5 people will not have flipped a single tail. They are the best coin flippers in the world.

Studies have shown that managers whose performances have been tracked have been unable to beat their benchmarks over extended periods of time (10+ years) to a statistical degree greater than the coin flippers (i.e. no better than what would be the expected random statistical result). And even if we were to assume for a moment that those that did, weren't just statistically lucky (coin flippers) but rather it was skill, the studies have shown that identifying those winners in advance was impossible.

And if there is anyone in this discussion who has won a Nobel Prize in economics, then tell me who and I'll gladly defer to him or her. But the people and principles I've been discussing HAVE been the result of several Nobel Prize winners in economics.
Most of all  
Bill2 : 6/1/2015 7:30 am : link
This is a dead end. If it works for you...go ahead.

When people point out that there are reasons it might not in the years ahead...The smart move is to ask: why?

Instead you are convinced there is nothing more to learn or consider and anyone who might o something you don't is flawed.

That's why you are getting attacked. A reasonable and fully informed and open to possibilities for further learning is not closed minded and does not attack messengers ...Instead listens harder to what the message might be.

Again...not an academic and not on wall street and a person who self teaches himself the old fashioned way....reading a variety of sources and listening to a variety of viewpoints. And from that perspective. ..my advice was to be open. Offered many opportunities to ask...what is possibly flawed about my approach. ..you doubled down.

Dismissed as not worth talking to on this subject. But I always liked your posts on the Giants and make sure to read them. So let's meet on better ground and leave this one to unfortunate.

Take care.
Read your last post after I was writing mine  
Bill2 : 6/1/2015 7:42 am : link
Look this is ships passing in the night.

No one no one no one debates the math of an argument about investing around since the 1950s.

The 1950s. The 1950s.

Your new found nugget is very old and very understood. No one is arguing about the mechanics of the theory. It's math. No one can.

What people are telling you is that the correct "truths" of the earlier era of the way this asset class was traded may no longer apply. MAY. MAY.

Bill ..let's back away from the thread for two days. Clear your mind. You are acting like anyone is debating the details of the approach. You don't need a weatherman to know which way the wind blows. The approach is ages old and BBGOTFO. Accept that. It is. It's obvious what you are saying is correct for a cyclical market in a cyclical economy. WE VIOLENTLY AGREE.

you don't need to go on endlessly. It's not new news to anyone. Believe me. It's not a nugget.

That's not what people are trying to help you understand. ..but you lost anyone's interest in trying to show you the POSSIBLE limits of the approach.

But you know best.
RE: Most of all  
baadbill : 6/1/2015 7:46 am : link
In comment 12309046 Bill2 said:
Quote:
This is a dead end. If it works for you...go ahead.

When people point out that there are reasons it might not in the years ahead...The smart move is to ask: why?

Instead you are convinced there is nothing more to learn or consider and anyone who might o something you don't is flawed.

That's why you are getting attacked. A reasonable and fully informed and open to possibilities for further learning is not closed minded and does not attack messengers ...Instead listens harder to what the message might be.

Again...not an academic and not on wall street and a person who self teaches himself the old fashioned way....reading a variety of sources and listening to a variety of viewpoints. And from that perspective. ..my advice was to be open. Offered many opportunities to ask...what is possibly flawed about my approach. ..you doubled down.

Dismissed as not worth talking to on this subject. But I always liked your posts on the Giants and make sure to read them. So let's meet on better ground and leave this one to unfortunate.

Take care.


Bill, I understand and don't disagree with most of your observations. I knew what I was getting into. The subject is a heated one for me because of (a) the amazing lack of public education on personal finance; and (b) the intentional misinformation by Wall Street.

At the end of the day I should know better than to involve myself publicly on a bulletin board on a subject for which I have such strong feelings. I know I should walk away. A flaw of mine that drags me deeper and deeper.
I'll second what Bill says  
WideRight : 6/1/2015 7:50 am : link
Your points are abstractions from your preferred books or websites that anyone can evaluate on their own. Expecting affirmation from those you criticize is foolish.

If your pearls where really that valuable, a wise man would keep it to himself and make millions off of it. Go kill it. Good luck.


Badbill  
Bill2 : 6/1/2015 8:16 am : link
I know you from many a reasoned discussion on other threads. I do completely agree that it is almost a tragedy how uninformed most citizens are on personal finance. Understand passion behind the wrecks and suboptimal results which come from a lack of good perspective.

I don't think WS is any less immune from the sin of certainty. Which befalls academics, doctors, lawyers and Indian chiefs. And all humans.

So I don't think most on WS are deliberately doing anything. I am often comfortable amongst the top people in that field for they are at he end of the day pragmatic and open to what works and open to the exceptions which prove the rule and facile with layered second and third derivative problem solving. I cant stand the lesser players.

As for the FIRE institutions ( not people...institutions)....I have contempt and anger at the wreckage left of our nations potential for next generations for I think our vital institutions ( wisely preserving and wisely investing the nations wealth for the future is vital) are terribly misaligned to the national purpose and internally their "performance metrics" and rewards are very badly misaligned to optimal sound outcomes. Add the inability and disinterest to quality control as a check against greed and misalignment of their own metrics and rewards and we have a national crisis which is unwinding our nation.

Which ties to what I am trying to tell you. The approach is sound if the asset class is being traded as it use to be. Does that mean that WS advice is better under the new possibilities? Not at all. Not remotely saying that.

Simply put...the accumulated consequences of accelerating misalignment allowed by purchased politicians and a repurposed towards economic fascistic representation of the will of the people combined with accelerating internal misalignment of the institutions MAY have led us to a place where the asset class has to be preserved in very close to upwards only mode if we are to maintain some semblance of a social contract for the decades ahead. And that strategy may work or not. If its not successful...or they do succeed in keeping the asset class a bulwark of polity cooperation when the elite mess up...cyclicality is minimal in the years ahead. Now that may mean that getting into the asset class at any time is clever....I would not know. My own view is that investing without owning to the point of control is clever but never wise.

lets face it...all business is about clever and not about wise.
RE: Badbill  
baadbill : 6/1/2015 8:32 am : link
In comment 12309080 Bill2 said:
Quote:
I know you from many a reasoned discussion on other threads. I do completely agree that it is almost a tragedy how uninformed most citizens are on personal finance. Understand passion behind the wrecks and suboptimal results which come from a lack of good perspective.

I don't think WS is any less immune from the sin of certainty. Which befalls academics, doctors, lawyers and Indian chiefs. And all humans.

So I don't think most on WS are deliberately doing anything. I am often comfortable amongst the top people in that field for they are at he end of the day pragmatic and open to what works and open to the exceptions which prove the rule and facile with layered second and third derivative problem solving. I cant stand the lesser players.

As for the FIRE institutions ( not people...institutions)....I have contempt and anger at the wreckage left of our nations potential for next generations for I think our vital institutions ( wisely preserving and wisely investing the nations wealth for the future is vital) are terribly misaligned to the national purpose and internally their "performance metrics" and rewards are very badly misaligned to optimal sound outcomes. Add the inability and disinterest to quality control as a check against greed and misalignment of their own metrics and rewards and we have a national crisis which is unwinding our nation.

Which ties to what I am trying to tell you. The approach is sound if the asset class is being traded as it use to be. Does that mean that WS advice is better under the new possibilities? Not at all. Not remotely saying that.

Simply put...the accumulated consequences of accelerating misalignment allowed by purchased politicians and a repurposed towards economic fascistic representation of the will of the people combined with accelerating internal misalignment of the institutions MAY have led us to a place where the asset class has to be preserved in very close to upwards only mode if we are to maintain some semblance of a social contract for the decades ahead. And that strategy may work or not. If its not successful...or they do succeed in keeping the asset class a bulwark of polity cooperation when the elite mess up...cyclicality is minimal in the years ahead. Now that may mean that getting into the asset class at any time is clever....I would not know. My own view is that investing without owning to the point of control is clever but never wise.

lets face it...all business is about clever and not about wise.


Bill, that analysis is way above my pay grade. I do agree about the dangers - John Bogle has a fascinating book about how institutional ownership of stocks has risen from less than 5% to closer to 80%+ today - and since mutual funds and pension funds don't vote their shares - and have their own self interests - the result is that the traditional oversight of corporate management (stockholders) has been structurally changed (eliminated) - leaving CEOs and others to pillage public corporations for their own short term gain at the expense of the long term interests of stockholders (i.e. you and me with our 401k investments).

Anyway, that is really quite a different subject than the one I was originally discussing - which was a simple one. If you are buying stocks regularly - and can buy them at a lower price this week than last week, that is a good thing in the long run. Whether or not that is possible is an entirely different story.

But I do appreciate your overall advice and should probably "tone things down". Thanks Bill,

Bill
Thats giving him too much credit  
WideRight : 6/1/2015 8:38 am : link
He started out with an irrelevant hypothetical and used it to insult others, and now claims thats its really about his concern for how others (who he insulted) invest their money. And of course the greedy WS boogeymen...

And Bill, there was nothing simple about that last paragraph..
RE: Thats giving him too much credit  
baadbill : 6/1/2015 8:51 am : link
In comment 12309101 WideRight said:
Quote:
He started out with an irrelevant hypothetical and used it to insult others, and now claims thats its really about his concern for how others (who he insulted) invest their money. And of course the greedy WS boogeymen...

And Bill, there was nothing simple about that last paragraph..


That's simply not true. I could care less how people who work on Wall Street invest their money. As for the "greedy WS boogeymen" - go ahead and invest your money with them and good luck to you. It's no skin off my back.
It sounds like what you guys are saying is:  
idiotsavant : 6/1/2015 9:18 am : link
"apply lessons learned in bulletin board high volume or closely held mini stocks to the market at large - all assets"

(which might be to say - ''if you cannot move the asset up or down yourself, or know someone, - watch out'')

Which might be only partially true, actual demand or lack of, for things like iron, still probably has input.

On TV recently there was a (big iron trader?) being called out for (getting caught long?) when demand for steel was not really pulling iron along, and even that huge pool of un-deployed money was not enough to pull it up.

Again, shooting arrows in the dark here, but the idea that assets can be forced into a ever ending upwards trend might be a bit too much without any real demand increase?

In any case, I don't care really, to me, taking the market as a means and an end in and of itself, without talking about the real economy (real jobs and wages) seems a bit irresponsible whichever position you take on investing.
.....  
ctc in ftmyers : 6/1/2015 10:39 am : link
"Anyway, that is really quite a different subject than the one I was originally discussing"

But that is what the original OP was about and a good discussion was being had.

You came in and high jacked the thread. Several posters tried to steer you in the direction of what was being discussed and you took offense.
Fairly pathetic.  
manh george : 6/1/2015 11:00 am : link
He doesn't even know what academic literature is.

Charles Ellis is a smart guy, with a very specific objective and agenda. What his book IS NOT is academic literature.

Ignorant + smug + self-important + not open to the ideas of others sounds more like a fundamentalist (any religion) than a conveyor of knowledge. My favorite review on Amazon of his buddy Charlie's recent book:

Quote:


So bill, could you give us an example of actual academic literature you have read, since you claim that is your source?
Hoary, overblown humble-bragging
By Rich Stevens on May 14, 2015
Format: Kindle Edition
The contents of this book could have been summed up in one page, or perhaps even one sentence: invest in a weighted portfolio of index funds. Ok, we get it. And we get that Ellis is on the board of Vanguard, so he stands to personally benefit financially a great deal from that advice.

But do we have to endure his endless humble-bragging? His intonations of Choate, Greenwich, ad nauseum? And he did not merely graduate from HBS in 1956- no, he graduated from "THE Harvard Business School." Maybe he needed to clarify because there are two of them- I don't know.

But for a true vomit-inducing round of humble bragging, look no further than the Preface: "A large English oak table dominating the inside left corner of the Morning Room on the ground floor of Boodles, the oldest of the social clubs established more than a century ago in or near St. Jame's in London, is one of the places in which parts of this book were written. Other locations include hotels rooms in- and airplanes flying between- Johannesburg, San Francisco, Chicago, Nairobi, Princeton, Bermuda, Vail, Boston, New York City, and Atlanta; and of course, at home in Greenwich."

Gag.

For extolling the benefits of indexing for the common man, he's hardly a "champion of the people" if you ask me.
hahaha, prolly a baby boomer as well  
idiotsavant : 6/1/2015 11:36 am : link
hahahaha
Invoking academics as an appeal to authority based on  
kicker : 6/1/2015 12:11 pm : link
journal research is questionable enough. The biases inherent in the literature is gag worthy enough (look at the whole rational expectations and perfectly efficient market hypotheses to see this).

But to use books as bases for assertions, when they are not peer reviewed and targeted for gullible laymen? Ok...
holy fuck people  
idiotsavant : 6/1/2015 12:21 pm : link
yahoo fucking finance. done.
What happened baadbill?  
Dan in the Springs : 6/4/2015 12:11 pm : link
No acknowledgement of my post? If I'm correct, it was my initial post that got you going on this thread. You said that I should be praying for and cheering when the market tanks. I responded with my real-world situation, and you haven't acknowledged it.

The way I see it your statement was too full of assumptions and you haven't convinced me otherwise. Care to respond?
Charles Ellis has more experience than me.  
manh george : 6/4/2015 12:33 pm : link
He's 77 and I'm only 67. So my 45 years of experience is inadequate, I guess. The thing is, most of the really smart people in the investment community are a lot younger than me, not a lot older.

Should we measure dick size next? It's just as useful a comparison.

The thing is, an awful lot of the Nobel Prize winning economists are stuck 10-20 years in the past. I recently attended a lecture by Solow from MIT, Noble in 87. He though that technological change was exclusively about robots--and he's at the same school with 2 of the best futurists on the planet.

And back to the assertion that over the long run, stocks always beat bonds, that's absolutely true--until the single time that it isn't.

At this phase of technological change, owners, including owners of shares, are winning, hence the rich valuations.
Look up secular stagnation as a modern topic if you are going to argue that this will always be the case.

It's like the husband in that awful Jennifer Lopez movie, "Enough."

"All I have to do is hit you once, baby." Same holds for an overly stock-heavy portfolio. All we need is one economic crisis that occurs while the central banks don't have any stimulative arrows in their quiver, or where output capacity runs too far ahead of demand as we move further into the environment of accelerating technological change, and that working assumption would fall apart.
best post ever  
idiotsavant : 6/4/2015 4:47 pm : link
and a good thread overall
Not one of you have bothered to refute what Charles Ellis says...  
baadbill : 6/5/2015 3:10 pm : link
but of course to refute it, you'd have to understand it ... and to think that some of you honestly believe you know more than Nobel Prize Award winners is pretty telling.

I suppose you know more than David Swenson too. I trust you know who he is. Swenson wrote the forward to Ellis' book and has his own books that say the same as Ellis. Warren Buffet. Says the same as Ellis. Peter Lynch. Too more idiots I suppose. Then there's John Bogle. Another idiot. I could go on but a football fan site just happens to have the worlds's greatest financial minds (who can't figure out why a falling market is a good thing for a buyer of stocks ... lol ... what a joke).
hmm  
WideRight : 6/5/2015 3:38 pm : link
Exerpt:

"So, CHEER when the market goes down and you are buying more shares with your weekly contribution. And Pray that next week stocks get cheaper still, so you buy even MORE shares. And my goodness, if you can possibly be fortunate enough to be contributing when there is an actual CRASH (say a 40-50% drop), borrow money on your house, beg borrow and steal, do whatever you can to BUY BUY BUY cheap stocks. And pray for it to go down even further."
Please please please  
Bill2 : 6/5/2015 4:17 pm : link
Can we acknowledge that any advice about the market pre the significant changes of 2008 and 2009 should be re examined? Just that question....nothing else for the moment? Is the "market" for the asset class operate meaningfully different than say 2000? Yes or no?

All must concede the arithmetic that says something bought that was cyclical in the preJIT, pre 2008 monetary policy environment will go up and down in the pre cyclical preJIT, pre HFT, pre hyper algo hadoop cluster enabled pattern recognition software world. Pre overnight window change world. Pre shorts taken out and bled every afternoon world. Ok?

Buying less for something that was in the past highly likely to go up made sense. Wow. Stop the presses. Of course that's true if you picked the right market basket.

So on that point you have measured with a micrometer. For that environment. ..that is stipulated as truth within this conversation.

Here is where you miss by a mile.

We are not in a pre cyclical, pre hadoop, pre overnight window change, pre new monetary policy change, pre hyper sensitive pattern recognition software world ...are we?

Fact. We are not.

So prior advice must be questioned. True for academics, idiots, football fans and people who are exposed to but retain and independent thinking processing mind dedicated to learning. For example almost but not all the people on the thread
For one thing  
Bill2 : 6/5/2015 4:43 pm : link
There are not shorts in the "market"

It's the asset class needed to preserve social order for all those who do not live on government wages contracts or transfer payments.

To Manh ' s point; is it structural or still cyclical?
So  
Big Al : 6/5/2015 4:55 pm : link
past performance is not necessarily indicative of future results?
eh  
Bill2 : 6/5/2015 5:04 pm : link
I thought it was closer to "past environment and characteristics and probability calculus " is not an indication of future environment, characteristics or payoff probability.

Take care Big Al. Ditto my friend on many a thread; Badbill
Better to keep it simple  
Big Al : 6/5/2015 5:11 pm : link
for economic dummies like most of us here.
Big Al  
Bill2 : 6/5/2015 5:42 pm : link
Just trying to distinguish market characteristics
Performance from "stock" performance.

but as an engineer I can see how your labeling is perfectly fine
This still going?  
manh george : 6/5/2015 7:04 pm : link
OK, let's deconstruct this sentence:

"Not one of you have bothered to refute what Charles Ellis says but of course to refute it, you'd have to understand it ... and to think that some of you honestly believe you know more than Nobel Prize Award winners is pretty telling."

1) "Understanding" is not the same as "knowing more than." How did you make that leap?

2) I would never claim to know more than Ellis...but I would claim, from time to time, to know more than what he writes in a book targeted at readers with roughly a 9th to 10th grade education.

3) What he rights in such a book isn't academic research.

4) As noted in one of the Amazon reviews, he is writing from a very specific viewpoint. I am not required to agree with that viewpoint, which in this case is that a)indexing is the only way to go, and b) every downturn is cyclical.

5) I disagree with Nobel Prize winning economists very frequently. The thing is, they frequently disagree with each other, so I am likely to have some other ones that agree with me.

6) There are tried and true techniques for deconstructing academic research, even if you don't understand all of the math underlying it. I do it frequently in my profession. I don't need that here, of course, because this isn't academic research. Or close to it.

7) There is a term for betting on dips, and then betting some more on a bigger dip, and so on. It's called a "Taleb Distribution."

Quote:
In economics and finance, a Taleb distribution is a returns profile that appears at times deceptively low-risk with steady returns, but experiences periodically catastrophic drawdowns. The term was coined by journalist Martin Wolf and economist John Kay, and is named after Nassim Taleb, based on ideas outlined in his Fooled by Randomness.[1]According to Taleb in Silent Risk, it should be called "payoff" not a "distribution".[2] It does not describe a statistical probability distribution, and does not have an associated mathematical formula. The term is meant to refer to an investment returns profile in which there is a high probability of a small gain, and a small probability of a very large loss, which more than outweighs the gains. In these situations the expected value is very much less than zero, but this fact is camouflaged by the appearance of low risk and steady returns. It is a combination of kurtosis risk and skewness risk: overall returns are dominated by extreme events (kurtosis), which are to the downside.


Try that on for size, genius.
Link - ( New Window )
building on Manhs point  
Bill2 : 6/5/2015 7:23 pm : link
In the structure of this market the phenomenon described as a small possibility of a very large loss....is much greater than the "market" when Taleb wrote. And the chances for "dips" are less.

Six traders dominate the market and are millisecond tuned to each other. Watch every afternoon necessary for sudden money to come into the market.


Do you think Goldman and Morgan can count on one hand the days of loss they have per quarter is like ...you know...The way a "market" pre 2005 worked?
manh george  
baadbill : 6/5/2015 7:42 pm : link
I can't disagree with you more completely.

Having said that, when you bring Taleb into the conversation, I'll concede your intellectual superiority to the extent you can read Taleb and fully comprehend on his level. Alas, I must confess, I cannot. I've read two of his books. I LOVE his incredible thinking but struggle to fully comprehend his writing (which is why I've read his books several times - each time I comprehend a bit more, but I'm still have a way to go).
RE: hmm  
baadbill : 6/5/2015 8:26 pm : link
In comment 12315958 WideRight said:
Quote:
Exerpt:

"So, CHEER when the market goes down and you are buying more shares with your weekly contribution. And Pray that next week stocks get cheaper still, so you buy even MORE shares. And my goodness, if you can possibly be fortunate enough to be contributing when there is an actual CRASH (say a 40-50% drop), borrow money on your house, beg borrow and steal, do whatever you can to BUY BUY BUY cheap stocks. And pray for it to go down even further."


LOL ... if there is anything I wrote here that makes the most logical sense - with which Buffet, Lynch, Bogle, Swensen and most finance experts would agree, you just quoted it.
baadbill  
ctc in ftmyers : 6/5/2015 8:32 pm : link
What happens when the fed starts easing and in interest rates go back up to 7%?

You are familiar with the rule of sevens aren't you?
RE: baadbill  
baadbill : 6/5/2015 8:51 pm : link
In comment 12316276 ctc in ftmyers said:
Quote:
What happens when the fed starts easing and in interest rates go back up to 7%?

You are familiar with the rule of sevens aren't you?


Huh? Fed starts easing?

Not sure what you are talking about. The point I made was a very simplistic one. It is better to buy stocks at $10 a share than at $20 a share.

Over a 60 year investment lifetime, the long term trendline for stock prices will be up (assuming GDP 60 years later is higher than it was 60 years earlier - and if it isn't we have a totally different and much bigger problem)... so, the trendline is going to be up.

During those 60 years, however, the year by year prices are going to fluctuate wildly and there will be times when prices are very high relative to earnings and other times when prices will be very low relative to earnings. And as a "buy and hold" investor the ups and downs of the stock market don't make any difference to you except for when you are making purchases. And for purchases, you'd like the purchases to be made on "down days". And since you aren't selling, market highs shouldn't really make you feel all that good - after all, you aren't selling and so the prices are fictional for your purposes anyway. The only price that matters to the buy and hold investor is the price when he/she finally sells during retirement - which almost certainly is going to be much, much higher than when he/she made the first purchase 40 years earlier - and hopefully more than the purchase made 10 years earlier too.

The real difference in portfolio performance for investors who have the same asset allocations and are indentical buy and holder investors, will be the luck of the market's performance over your investment lifetime and into retirement - what is known as the "sequence of returns" risk. Which is most often discussed in terms of stock returns during the initial years of retirement - but the concept applies equally well (but less discussed) over an investor's lifetime. A lucky investor is one who is fortunate enough to have had lots of market lows when he/she purchased most of his/her stocks - accumulating lots of shares - and then been fortunate enough to have the stock market go on an incredible bull run right when he/she retires (i.e. when selling those shares to live on).

Anyway - that's all I've been talking about since my first post. Pretty simple stuff. Going from there to Taleb's Black Swan is an entire different matter. One of the greatest books every written - and scary as hell (to the extent I'm truly able to comprehend his concepts).

RE: baadbill  
baadbill : 6/5/2015 8:54 pm : link
In comment 12316276 ctc in ftmyers said:
Quote:
What happens when the fed starts easing and in interest rates go back up to 7%?

You are familiar with the rule of sevens aren't you?


CTC, do you mean the Rule of 72?
RE: RE: baadbill  
ctc in ftmyers : 6/5/2015 9:23 pm : link
In comment 12316300 baadbill said:
Quote:
In comment 12316276 ctc in ftmyers said:


Quote:


What happens when the fed starts easing and in interest rates go back up to 7%?

You are familiar with the rule of sevens aren't you?



CTC, do you mean the Rule of 72?


basically yes.

If interest rates remain high enough, with all else being compatible, growth, etc., it allows all to make money on their money with little risk instead of investing in the market which few can afford to.

Stimulates saving and amassing wealth.

Simplistic concept.

.  
idiotsavant : 6/5/2015 9:32 pm : link
and the fat lady signs
RE: RE: RE: baadbill  
baadbill : 6/5/2015 9:40 pm : link
In comment 12316324 ctc in ftmyers said:
Quote:
In comment 12316300 baadbill said:


Quote:


In comment 12316276 ctc in ftmyers said:


Quote:


What happens when the fed starts easing and in interest rates go back up to 7%?

You are familiar with the rule of sevens aren't you?



CTC, do you mean the Rule of 72?



basically yes.

If interest rates remain high enough, with all else being compatible, growth, etc., it allows all to make money on their money with little risk instead of investing in the market which few can afford to.

Stimulates saving and amassing wealth.

Simplistic concept.


The "problem" with 7 percent interest rates is inflation is almost surely the cause of such high rates, so "real" returns are unlikely to be satisfactory from such an investment. But, I'm not sure what that has to do with stock market variations and the "sequence of returns".
She signs?  
manh george : 6/5/2015 9:40 pm : link
What, she's fat AND deaf?
RE: RE: RE: RE: baadbill  
ctc in ftmyers : 6/5/2015 9:54 pm : link
In comment 12316334 baadbill said:
Quote:
In comment 12316324 ctc in ftmyers said:


Quote:


In comment 12316300 baadbill said:


Quote:


In comment 12316276 ctc in ftmyers said:


Quote:


What happens when the fed starts easing and in interest rates go back up to 7%?

You are familiar with the rule of sevens aren't you?



CTC, do you mean the Rule of 72?



basically yes.

If interest rates remain high enough, with all else being compatible, growth, etc., it allows all to make money on their money with little risk instead of investing in the market which few can afford to.

Stimulates saving and amassing wealth.

Simplistic concept.




The "problem" with 7 percent interest rates is inflation is almost surely the cause of such high rates, so "real" returns are unlikely to be satisfactory from such an investment. But, I'm not sure what that has to do with stock market variations and the "sequence of returns".


It has as much to do with this thread as your rambling on has.

Nothing.

You have already admitted that what is being discussed is above your pay grade.

Read and learn instead of painting yourself in a corner and putting blinders on to as what is actually being discussed.
holy shit, belly laugh  
idiotsavant : 6/5/2015 9:54 pm : link
seriously, woke people up.

sings.

I cannot even type anymore,

best post now a belly laugh


that fat lady signs, hahaha
and here she is  
idiotsavant : 6/5/2015 9:56 pm : link
https://www.youtube.com/watch?v=k_cnlQmsScU
and gosh  
idiotsavant : 6/5/2015 9:58 pm : link
that is a classic
Back to the Corner